14 cdem neurologic

Page 1

Fall 2014

N eurolo gic Edition

In This Issue Lesson 1 Nontraumatic Acute Increased Intracranial Pressure . . . . . . . . . . . . . . . . 3 Disorders of acute elevations in intracranial pressure (ICP) offer a challenge for emergency physicians and require an understanding of the physiology behind the disorders, the diagnostic modalities, and the nuances of management. Patients with elevated ICP can rapidly deteriorate and are at significant risk for vision loss, brain herniation, and death.

Lesson 2 Evaluation and Treatment of Status Epilepticus. . . . . . . . . . . . . . . . . . . 11 Seizure patients can present in dramatic fashion and require a rapid and organized diagnostic assessment in parallel with early and aggressive anticonvulsant therapy. This lesson reviews the latest recommendations for pharmacologic therapy as well as the options for airway management and the special considerations for pediatric patients.

Lesson 3 Peripheral Neuropathies. . . . . . . . . . . . . . . . . . . . . . . . . . . . . . . . . . . . . . 19 Emergency physicians must be adept at differentiating benign from life-threatening causes of peripheral neuropathies and be prepared to initiate appropriate management regardless of the cause. In this article, Guillain-BarrĂŠ syndrome, botulism, and carpal tunnel syndrome are examined.

Lesson 4 Evaluation and Management of Stroke. . . . . . . . . . . . . . . . . . . . . . . . . . 27 Timely recognition and treatment are crucial for optimal outcomes in patients with stroke. Patient stabilization, a focused examination, appropriate testing, and development of a presumptive diagnosis are all the purview of emergency physicians. A delay in the diagnosis and necessary treatment can have long-lasting, devastating consequences for the patient and can result ultimately in litigation.


Critical Decisions in Emergency Medicine / Neurologic Edition

Contributors Arleen Allen, MD, and Bernard L. Lopez, MD, MS, FACEP, wrote “Nontraumatic Acute Increased Intracranial Pressure.” Dr. Allen is an emergency physician practicing in Maryland. Dr. Lopez is vice chair and professor of emergency medicine at Thomas Jefferson University Hospital, Department of Emergency Medicine, in Philadelphia, Pennsylvania. Julie J. Cooper, MD, and Michael A. Gisondi, MD, FACEP, wrote “Evaluation and Treatment of Status Epilepticus.” Dr. Cooper is an attending physician with Doctors for Emergency Service at Christiana Care Health System in Delaware. Dr. Gisondi is the residency program director and associate professor of emergency medicine at Northwestern University Feinberg School of Medicine, Chicago, Illinois. David Della-Giustina, MD, FACEP, FAWM, and Tristan Knutson, MD, wrote “Peripheral Neuropathies.” Dr. Della-Giustina is an associate professor, program director of the YaleNew Haven Emergency Medicine Residency Training Program, and chief of the Section of Education in the Department of Emergency Medicine at Yale School of Medicine in New Haven, Connecticut. Dr. Knutson is a major in the US Army and is the assistant program director for the Emergency Medicine Residency at the Madigan Army Medical Center in Fort Lewis, Washington. Brian E. Burgess, MD, FACEP, and Justin Stowens, MD, wrote “Evaluation and Management of Stroke.” Dr. Burgess is assistant professor in the Department of Emergency Medicine at Jefferson Medical College and assistant residency director and academic and research faculty in the Emergency Medicine Residency Program at Christiana Care Health System in Newark, Delaware. Dr. Stowens is senior emergency medicine resident in the Emergency Medicine Residency Program at Christiana Care Health System in Newark. Daniel A. Handel, MD, MPH, FACEP, served as content reviewer for these lessons. Dr. Handel is chief medical officer and an associate professor of medicine and pediatrics in the Division of Emergency Medicine at Medical University of South Carolina in Charleston, South Carolina.

2


Critical Decisions in Emergency Medicine / Neurologic Edition

Lesson 1

Nontraumatic Acute Increased Intracranial Pressure Arleen Allen, MD, and Bernard L. Lopez, MD, MS, FACEP

■ From the EM Model

Disorders of acute elevations in increased intracranial pressue (ICP) offer a challenge for emergency physicians and require an understanding of the physiology behind the disorders, the diagnostic modalities, and the nuances of management. Patients with elevated ICP can rapidly deteriorate and are at significant risk for vision loss, brain herniation, and death. This lesson will review three causes of nontraumatic increased ICP: idiopathic intracranial hypertension (formerly pseudotumor cerebri), shunt malfunction, and acute hydrocephalus. It will examine the major complications and some of the controversies in diagnosis and management, including the utility of neuroimaging in shunt malfunction and alterations in rapid sequence (RSI) intubation.

12.0 Nervous System Disorders 12.4 Hydrocephalus

Case Presentations

■ Objectives On completion of this lesson, you should be able to: 1. Discuss causes and presentations of nontraumatic increased intracranial pressure (ICP). 2. Describe the pathophysiology of increased ICP as it relates to the disease entities. 3. Detail the diagnosis of increased ICP and its different entities. 4. Discuss the role of neuroimaging in evaluating ventriculoperitoneal shunts. 5. Discuss management of increased ICP. 6. Discuss the major complication of idiopathic intracranial hypertension.

■ Case One A 35-year-old woman presents to the emergency department because she has been having intermittent headaches for 6 months to a year. The headaches usually resolve with acetaminophen, ibuprofen, or rest. She has noticed that in the past 3 months the headaches have been increasing in intensity, the medications are not as helpful, and she will occasionally get a ringing in her ears. What concerns her most; however, is that she is often seeing double and feels like there is a haze in the corners of her vision.

On physical examination, the patient is mildly obese, alert, and in no distress. Head is normocephalic and atraumatic. She has mild horizontal nystagmus with extraocular movements intact. Her neck is supple. Neurologically, the patient has intact cranial nerves, sensation, and cerebellar function, with full strength throughout. Results of her laboratory evaluation, including a CBC and electrolyte panel, are within normal limits. A urine pregnancy test is negative. A noncontrast computed tomography (CT) scan of the head shows no intracranial hemorrhage or mass effect. The patient is given 1 liter of intravenous normal saline, prochlorperazine, and ketorolac, with subsequent resolution of her headache. She is discharged home with a diagnosis of headache of unknown origin and told to followup with her primary physician. ■ Case Two A 54-year-old man with a history of subarachnoid hemorrhage (SAH) complicated by hydrocephalus presents to the emergency department because he “passed out.” He had a ventriculoperitoneal (VP) shunt placed 2 years ago. He reports that 1 hour prior to presentation he was walking to the bathroom when he felt a posterior headache, not as intense as when he had SAH, but “pretty bad.” He called out to his daughter and slid to the floor, landing on his buttocks.

3


Critical Decisions in Emergency Medicine / Neurologic Edition

Critical Decisions • What findings suggest increased ICP? • How useful are shunt series and brain computed tomography (CT) in assessing patients with ventriculoperitoneal shunt malfunctions?

He did not hit his head and had no loss of consciousness; the headache is now lessening in intensity. On physical examination, the patient is alert and resting comfortably. Vital signs are blood pressure 148/85, pulse rate 75, respiratory rate 16, and temperature 36.6°C (97.9°F). The head is normocephalic and atraumatic. The neck is supple. Mucous membranes are dry. Breath sounds are clear. Heart sounds are normal, without murmur, rub, or gallop. The patient is alert and oriented to person, time, and place; cranial nerves II through XII are intact, without focal neurologic deficits. Motor strength is 5/5 throughout, sensation is intact to light touch, and cerebellar function is intact. Results of the laboratory evaluation, including a CBC and electrolyte panel, are normal. An ECG is unremarkable, with normal sinus rhythm and no ischemic changes. A noncontrast CT of the head shows an unchanged enlarged ventricle size, no intracranial hemorrhage, and no mass effect. A shunt survey shows no disconnection, migration, or kinking of tubing. After receiving 1 liter of normal saline intravenously and acetaminophen, 1,000 mg, the patient reports that his headache has resolved. He has called his wife to pick him up and wants to leave the emergency department. ■ Case Three A 51-year-old man with a history of hypertension and squamous cell carcinoma of the tongue and larynx who recently had surgical

• What role do lumbar punctures, shunt taps, and ventriculostomy have in the emergent management of increased ICP? • How does the management of idiopathic intracranial hypertension differ from the management of other causes of increased ICP?

resection, chemotherapy, and radiation is brought in via ambulance accompanied by his wife and son. For the past 2 weeks the patient has had fever and headaches. He was seen by his family physician who prescribed an unknown antibiotic and analgesics. For the past few days he has not been acting like himself and his speech has been incoherent; today he stopped speaking. This morning he was found lying on the floor after an unwitnessed fall. On physical examination, the patient is awake, alert, and nonverbal. Vital signs are blood pressure 230/95, pulse rate 90, respiratory rate 12, and oral temperature 38.9°C (102°F). The head is normocephalic and atraumatic. Pupils are equal, round, and reactive to light. There is minimal tracking for extraocular movements. The patient does not follow commands but is moving all extremities, with strength testing 3/5 throughout. Laboratory evaluation is significant for a WBC count of 39,000 and lactate of 30 mg/dL (normal is 4.5-19.8 mg/ dL). The electrolyte panel results are within normal limits. A noncontrast CT of the brain shows new hydrocephalus with dilation of the lateral ventricles, the third ventricle, and possibly the fourth ventricle. During the course of the emergency department evaluation, the patient has further neurologic deterioration, becoming unresponsive. An understanding of changes in ICP stems from an understanding of intracranial mechanics. The brain, blood, and cerebrospinal fluid (CSF) are housed in the fixed spaces of the

cranium and vertebral column. Any increases in the volume of the brain, blood, or CSF causes an increase in the ICP. Pressures greater than 25 mm Hg are considered “increased,” although adequate cerebral perfusion can be maintained at an ICP of up to 40 mm Hg if blood pressure remains normal.1 If CSF flow is impaired within the ventricular system (eg, mass lesions, stenotic lesions, or intraventricular hemorrhage), this can result in hydrocephalus, defined as an increase in volume and subsequent pressure. Alternatively, hydrocephalus can also occur with CSF impairment outside the ventricular system at the level of the arachnoid granulations or basal cisterns (eg, resulting from bacterial meningitis, SAH, spinal cord tumors). The most common cause of acute hydrocephalus is infection, most often bacterial meningitis.2 It often occurs several weeks after initial presentation but has been known to develop within days. Cerebellar encephalitis and neurocysticercosis can also be infectious causes. The second most common cause of hydrocephalus is intracranial hemorrhage, most often subarachnoid hemorrhage. Finally, idiopathic intracranial hypertension (also known as pseudotumor cerebri) is the term used to describe increased ICP in the absence of an identifiable cause such as tumor or infection. Idiopathic intracranial hypertension can have various etiologies or associations including medications such as tetracycline and retinoids, vitamin A toxicity, and diseases of endocrinology, including thyroid disease. There are several theories 4


Critical Decisions in Emergency Medicine / Neurologic Edition

explaining the development of this entity; a common theory is that obstruction of flow through the arachnoid villi or venous sinuses causes an increase in pressure without an increase in volume.3 CRITICAL DECISION What findings suggest increased ICP?

How a patient presents clinically depends both on the patient’s age and the mechanism of the intracranial hypertension. Infants with acute hydrocephalus often present with head enlargement greater than 97th percentile, bulging or tense fontanelles, nausea and vomiting, poor feeding and failure to thrive, failure of upward gaze (“sunset eyes”), irritability, and a depressed level of consciousness. In a study by Piatt and Gorton,4 bulging fontanelles had the highest association with increased ICP in the setting of shunt malfunction, followed by depressed level of consciousness and irritability. Furthermore, the absence of irritability, headache, and nausea/vomiting greatly reduces the likelihood of shunt failure. In children and adults, clinical manifestations of increased ICP include headache, nausea and vomiting, sleepiness, confusion, ataxia, ocular palsies, tinnitus, and papilledema. The Cushing triad of hypertension, bradycardia, and irregular respirations is a late finding indicating impending herniation or marked intracranial hypertension. Decerebrate or decorticate posturing is an even later sign. In idiopathic intracranial hypertension, the classic patient is an obese woman of childbearing age who presents with headaches and visual symptoms. However, it can affect children, men, and thin individuals. The headache varies considerably and can be a frontal, retro-ocular, or pressure headache, and it can have migrainous features (throbbing, photophobia, phonophobia, nausea) or a presentation suggestive of SAH (worst headache of one’s life). Vision

obscurations (partial or complete visual loss lasting seconds at a time) occur in 75% of patients and tinnitus in 50%. But the hallmark of the condition is papilledema, which is often bilateral but can be unilateral.5 The physical examination should include an assessment of the external head to rule out suspected traumatic causes of elevated ICP. The eyes must be examined thoroughly, including assessing pupil reactivity, shape, and equality; extraocular movements should be checked for palsies and visual field constrictions; and finally the fundi should be examined for papilledema. A thorough neurologic examination should be performed, including assessment of mentation, cranial nerves, motor and sensation, and cerebellar function. In lifethreatening situations, the evaluation should be narrowed to an assessment of mental status using the Glasgow Coma Scale (GCS) and a pupillary assessment.6 Laboratory evaluation is nonspecific for the etiology or diagnosis of elevated ICP. A noncontrast CT of the brain is the initial imaging recommended. It is helpful in examining ventricle size, intracranial hemorrhage, masses, and mass effect. In the case of shunt malfunction, a radiographic shunt series can reveal shunt migration, disconnection, or kinking. The shunt series comprises views of the anterior, posterior, and lateral skull; anterior and posterior chest; and anterior and posterior abdomen. CRITICAL DECISION How useful are shunt series and brain CT in assessing patients with shunt malfunctions?

In a retrospective study of 280 children with shunts presenting to a Toronto emergency department, Mater and colleagues7 looked at the utility of shunt series and brain CT in identifying shunt malfunction by correlating these studies with subsequent shunt revision. They found that abnormal shunt series readings had a sensitivity of 30%,

a specificity of 95.8%, a positive predictive value (PPV) of 72.3%, and a negative predictive value (NPV) of 75.1%. Abnormal brain CT readings had a sensitivity of 61%, a specificity of 82.7%, a PPV of 64.5%, and an NPV of 80.5%. These findings suggest that neuroimaging, which is routinely performed to exclude life-threatening complications, is insensitive and has poor predictive value. Zorc and colleagues8 retrospectively studied 140 children with shunts presenting to a Philadelphia emergency department, looking at the shunt series, brain CT, and both in combination. Results again showed poor sensitivity (20% in shunt series, 83% in brain CT, and 88% in combination) and predictive value (75%, 55%, and 55% respectively). In adults, Griffey and colleagues9 performed a retrospective review of 192 patients presenting to a Boston emergency department. Again, these studies were found to have poor sensitivity (11% shunt series, 52% brain CT, and 57% combination) and predictive value (56%, 33%, and 33%, respectively). In both children and adults, both shunt series and brain CT, in isolation or in combination, have been shown to have poor correlative characteristics with shunt malfunction. These findings bring into question whether these expensive and potentially harmful imaging studies are useful for emergency physicians. However, evaluating a shunt clinically is often difficult as well. Isaacman and Poirer10 presented three case studies of children with shunts with varied symptoms of headache, vomiting, and neck stiffness and asked three emergency physicians to discuss management. They conclude that results of the history and physical examination are often inconclusive. Despite various studies that have looked at clinical findings in relation to shunt malfunction, no set of clinical factors reliably predicts shunt malfunction.10 Thus, because both clinical findings and radiological studies are unreliable 5


Critical Decisions in Emergency Medicine / Neurologic Edition

in identifying shunt malfunction, neurosurgical consultation should be sought when there is persistent clinical concern despite a negative workup.

Management In general, once the diagnosis of elevated ICP has been made, the goal of management is to prevent secondary insult (eg, herniation and death in acute shunt malfunction and hydrocephalus and vision loss in idiopathic intracranial hypertension). In milder cases (headache, normal level of consciousness, no neurologic deficit), management centers on analgesia and diagnostic studies. In more severe cases in which consciousness is altered or neurologic deficits are present, airway control and ventilator management must be considered. Early endotracheal intubation and controlled ventilation are important— hypoxia and hypercarbia adversely affect cerebral perfusion pressure. Induction agents are important even in patients with GCS scores lower than 8, because induction agents blunt hemodynamic fluctuations that come with laryngoscopy and intubation. Meyer and colleagues6 recommend RSI with thiopental and succinylcholine/rocuronium in the hemodynamically stable patient and etomidate/ketamine and succinylcholine/rocuronium in unstable patients. Thiopental, propofol, and etomidate all reduce ICP. Although succinylcholine can cause a transient increase in ICP, the clinical relevance of this has not been established, and its use is not contraindicated.11 Furthermore, Clancy et al in a literature search could find no evidence that succinylcholine caused a detectable change in ICP in patients with acute trauma or bleeds.12 There is conflicting evidence about whether lidocaine should be used as pretreatment in RSI for patients with elevated ICP. Lidocaine is a fast sodium channel blocker whose exact mechanism for preventing increases

in blood pressure is unclear, but when it is given at a dose of 1.5 mg/ kg it is known to suppress the cough reflex, preventing elevation in ICP. One set of authors13 opposes its use given the lack of direct evidence (its use is often inferred from studies with tracheal suctioning rather than endotracheal intubation). Moreover, they note that lidocaine takes several minutes to provide its blunting effect, which can be deleterious in a rapidly decompensating patient who needs an emergent airway. And, when it does take it effect, it can cause a sustained decreased in mean arterial pressure and subsequent perfusion pressure. Caro and Bush continue to recommend its use based on its cough suppression effect and especially when no neuromuscular blockade is given.14 Fentanyl, 1 to 3 mcg/kg, is also recommended as possible pretreatment to blunt ICP. Routine use of atropine in children and a defasciculating small dose of a competitive neuromuscular blocker in conjunction with succinylcholine administration are no longer recommended. With respect to ventilation, there is debate about the utility of hyperventilation therapy in decreasing ICP. Hyperventilation reduces ICP by causing vasoconstriction and a reduction in cerebral blood flow, which in turn can cause ischemic consequences. A Cochrane review15 concluded that data are inadequate to assess either potential harm or benefit. A recent neurosurgical workgroup16 details guidelines for using hyperventilation in elevated ICP. It should not be used routinely or prophylactically. If hyperventilation is used, it should only be a temporizing measure, but not in the first 24 hours after intubation, as the brain is more subject to insult during that period. To further decrease ICP, mannitol or hypertonic saline solution is used. Mannitol is given in a 20% rapid infusion. It acts by inducing water transfer and decreasing total brain water content and volume. Mannitol

also acts by a reflex vasoconstriction to reduce ICP. CRITICAL DECISION What role do lumbar punctures, shunt taps, and ventriculostomy have in the emergent management of increased ICP?

All of these procedures act to measure and reduce ICP. The lumbar puncture can be a fatal procedure in a patient with increased ICP because of the risk for cerebral herniation. It should only be performed in neurologically intact patients with no papilledema and a brain CT showing no mass effect. There are four radiographic findings indicative of increased ICP: midline shift, obstructive hydrocephalus, compression of basilar cisterns, and compression of fourth ventricle.17 The lumbar puncture is useful in chronic VP shunt patients with communicating hydrocephalus and concern for meningitis. Patients with VP shunts can have compartmentalization of CSF, allowing infection to occur in the ventricles but not in the meninges, and vice versa. With communicating hydrocephalus, pressures between intracranial compartments are equal and lumbar puncture is thought to be safe. However, herniation has occurred in patients with meningitis who had “normal� CT scans.17 Therefore, following lumbar puncture, such patients should be closely monitored for signs of neurologic deterioration. The lumbar puncture is also the definitive diagnostic modality in idiopathic intracranial hypertension; CSF analysis will be normal except for an elevated opening pressure (above 20 cm H2O in a thin patient, above 25 cm H2O in an obese patient). A shunt tap is a percutaneous needle aspiration of ventricular CSF. It is indicated in patients with VP shunts and concern for ventriculitis, and more importantly it is decompressive therapy in the setting of increased ICP. The main risks of the procedure are introduction

6


Critical Decisions in Emergency Medicine / Neurologic Edition

of infection, brain edema, and intraparenchymal bleeding. The first can be prevented by strict aseptic procedure, the last two by avoiding rapid removal of CSF. Ventriculostomy is indicated for the critical, rapidly declining patient with increased ICP from acute hydrocephalus or massive intraventricular hemorrhage. It offers a temporizing measure for diversion of fluid or blood until definitive surgical intervention can take place. Its major complication is infection, most often secondary to skin flora contamination. CRITICAL DECISION How does the management of idiopathic intracranial hypertension differ from the management of other causes of increased ICP?

Once the diagnosis of idiopathic intracranial hypertension has been made (ie, elevated opening pressure with normal neuroimaging), the goal of management is to preserve vision. Papilledema can occur early in the course of idiopathic intracranial hypertension—even before the patient becomes symptomatic with visual obscurations.5 Papilledema is often difficult to detect with direct ophthalmoscopy, and ophthalmology consultation is often needed. Visual changes include constriction of field (central vision is usually involved late), blind spots, diplopia, and blurred vision. The major concern is that papilledema can lead to optic nerve atrophy and blindness. Spontaneous recovery is common (70%), but optic nerve atrophy and blindness can also recur (10% in 1 year, 40% in 6 years, 15% in 13 years).3 Blindness occurs in 5% to 10%—unilateral or bilateral. The risk of blindness is related to the duration of papilledema and the rate of developing optic atrophy. Rarely does blindness occur within a few days.3 Once emergencies such as meningitis and subarachnoid hemorrhage have been excluded, a patient with idiopathic intracranial hypertension can be managed

in an outpatient setting. Medical management includes weight loss counseling, acetazolamide, and corticosteroids. Acetazolamide is first-line drug treatment. It is a carbon anhydrase inhibitor that acts to reduce CSF production and thus ICP. Furosemide can be an adjunct if the effects of acetazolamide are insufficient or the side effects are intolerable. Oral corticosteroids are often considered in the short term (1 to 2 mg/kg for 2 to 6 weeks) for acute severe cases, but there is a risk of weight gain. Surgery (shunt procedure or optic nerve fenestration) is indicated when visual deterioration and disabling headaches persist despite medical treatment. It is not recommended for headaches alone. Shunting is usually lumboperitoneal rather than ventriculoperitoneal. Shunting carries the risk of overdrainage, which can result in low-pressure headaches, tonsillar herniation, lumbar radiculopathy, and shunt failure. Optic nerve sheath fenestration is thought to work by immediate lowering of the CSF pressure in the subarachnoid space surrounding the optic nerve. Later, scarring of the fenestration can protect against transfer of the increased ICP from the sheath to the optic nerve. Most patients (two thirds) will achieve initial relief of headache and improvement or stabilization of visual function. There is a risk; however, of later deterioration of visual function and central retinal artery occlusion.3 Emergency department management is based on symptoms. If patients are asymptomatic, they should be educated and referred to a specialist (neurology or ophthalmology). Symptomatic headache patients should be given analgesics, and initiation of acetazolamide should be considered in consultation with a neurologist or ophthalmologist. Patients with severe or rapid optic neuropathy should be admitted with specialist consultation and attempts at reduction of ICP (acetazolamide, mannitol).

Case Resolutions ■ Case One One month later, the 35-year-old woman returned with worsening headaches and vision changes. The morning of this visit she noticed that she could not see anything for several seconds before her vision returned. She made an appointment to see her primary physician, but he was not available for 2 months. On this visit, the emergency physician detected papilledema on ophthalmoscopic examination. A lumbar puncture was performed and revealed an opening pressure of 28 cm H2O, but results of CFS analysis were otherwise normal. The on-call neurologist was consulted by phone, and the patient was started on topiramate for analgesia and acetazolamide and given an appointment to see the neurologist in 2 days for further neuroimaging. On discharge, the patient’s headache had improved and her visual obscurations had not returned. She was given instructions on idiopathic intracranial hypertension, counseled on the benefits of weight loss, and informed of the risk of permanent visual changes and possible loss of vision with her condition. ■ Case Two The 54-year-old man agreed to wait until the emergency physician could discuss the case with his neurosurgeon. The neurosurgeon had several concerns—syncope, shunt malfunction, and new subarachnoid hemorrhage. The patient and his wife agreed to hospital admission. Results of a cardiac workup (serial cardiac enzymes, echocardiogram, and neuroimaging [including magnetic resonance imaging, magnetic resonance angiography, and magnetic resonance venography of the brain and neck]) were normal. The VP shunt was tapped and revealed normal opening pressure and CSF analysis; a shunt revision was not performed. The patient was discharged home 2 days later with a diagnosis of vasovagal syncope. 7


Critical Decisions in Emergency Medicine / Neurologic Edition

Pearls • Cushing triad is a late sign of elevated ICP. • The hallmark of idiopathic intracranial hypertension is papilledema. • The shunt series and brain CT have been shown over several studies to have poor sensitivity and predictive value for identifying shunt malfunction. • Even in the obtunded patient, rapid sequence intubation should be performed to blunt elevations in ICP. • Routine atropine and defasciculating doses of a competitive paralytic are no longer recommended to blunt ICP elevation. • Hyperventilation should not be used routinely or prophylactically in elevated ICP, and it should never be used acutely in the first 24 hours.

Pitfalls • Not doing an ophthalmoscopic examination. • Assuming that negative shunt series and negative brain CT rule out shunt malfunction; half of patients with a shunt malfunction requiring shunt revision will not have a positive test. • Not involving specialists when clinical concern persists for shunt malfunction or idiopathic intracranial hypertension. • Performing a lumbar puncture in a patient with increased ICP can result in cerebral herniation.

■ Case Three The 51-year-old man was presumed to have acute obstructive hydrocephalus secondary to meningitis. Rapid sequence intubation was performed with etomidate and succinylcholine. No lidocaine or fentanyl was used for pretreatment. No hyperventilation was performed. The patient was started on broad-spectrum antibiotics. Neurosurgery was called for emergent ventriculostomy for decompression and CSF analysis. The opening pressure was more than 50 cm H2O, and CSF analysis revealed a WBC count of 540 with a neutrophilic predominance of 82%, an RBC count of 6, normal glucose, and normal protein. In the neurosurgical ICU, the patient, who had initially been hypertensive, became hypotensive, requiring vasopressors. Magnetic resonance imaging of the brain showed extensive leptomeningeal, parenchymal, and bone enhancement with concern for ventriculitis, osteomyelitis of the skull, and multiple areas of infarction in the cerebellum and midbrain. Two sets of blood cultures grew out after only 2 days with Streptococcus intermedius. Despite antibiotics guided by consultation with an infectious disease specialist and vasopressor support, the patient’s neurologic status and prognosis continued to remain poor. The patient was placed under comfort care by his family, and 2 days after presentation to the emergency department he died.

Summary Acute elevations in ICP can cause significant morbidity and mortality. Emergency physicians must have an understanding of the different diagnostic modalities and their utility. An ophthalmoscopic examination is vital in diagnosis and should not be forgotten in the workup of patients with increased ICP. A noncontrast brain CT is the appropriate imaging starting point. Specialist consultation and involvement are important, especially when clinical

concern persists despite negative neuroimaging or negative direct ophthalmoscopy. Emergency physicians must also be aware of the alterations in management with respect to RSI, ventilation, and decompressive therapy in patients with elevated ICP. There is on-going debate in the literature about pretreatment with lidocaine; but succinylcholine is considered safe, and hyperventilation is no longer recommended in acute management. The lumbar puncture is to be deferred with findings of mass effect on neuroimaging. Finally, for patients with confirmed or suspected idiopathic intracranial hypertension, appropriate, close, outpatient followup arrangements should be made.

References 1. Ropper AH, Brown RH. Disturbances of cerebrospinal fluid and its circulation, including hydrocephalus, pseudotumor cerebri, and low-pressure syndromes. In: Adams and Victor’s Principles of Neurology. 8th ed. New York, NY: McGraw-Hill; 2005:529-545. 2. Nasr FF, Honeycutt JH. Management of acute hydrocephalus. In Loftus CM, ed: Neurosurgical Emergencies. 2nd ed. New York, NY: Theime; 2008:27-33. 3. Skau M, Brennum J, Gjerris F, Jensen R. What is new about idiopathic intracranial hypertension? An updated review of mechanism of treatment. Cephalalgia. 2006;26:384-399. 4. Piatt JH, Garton HJL. Clinical diagnosis of ventriculoperitoneal shunt failure among children with hydrocephalus. Pediatr Emerg Care. 2008;24(4):201-210. 5. Friedman DI. Idiopathic intracranial hypertension. Curr Pain Headache Rep. 2007;11:62-68. 6. Meyer PG, Ducrocq S, Carli P. Pediatric neurologic emergencies. Curr Opin Crit Care. 2001;7:81-87. 7. Mater A, Shroff M, Al-Farsi S, et al. Test characteristics of neuroimaging in the emergency department evaluation of children for cerebrospinal fluid shunt malfunction. CJEM. 2008;10:131-135. 8. Zorc J, Krugman S, Ogborn J, Benson J. Radiographic evaluation for suspected cerebrospinal fluid shunt obstruction. Pediatr Emerg Care. 2002;18:337-340. 9. Griffey R, Ledbetter S, Khorasani R. Yield and utility of radiographic “shunt series” in the evaluation of ventriculo-peritoneal shunt malfunction in adult emergency patients. Emerg Radiol. 2007;13:307-311. 10. Isaacman DJ, Poirier MP, Hegenbarth M, et al. Ventriculoperitoneal shunt management. Pediatr Emerg Care. 2003;19:119-125. 11. Danzl DF, Vissers RJ. Tracheal intubation and mechanical ventilation. In: Tintinalli JE, Kelen GD, Stapczynski JS, eds. Emergency Medicine: A Comprehensive Study Guide. 6th ed. New York, NY: McGraw-Hill; 2004:108-119. 12. Clancy M, Halford S, Walls RM, Murphy M. In patients with head injuries who undergo rapid sequence intubation using succinylcholine, does pretreatment with a competitive neuromuscular blocking agent improve outcome? A literature review. Emerg Med J. 2001;18(5):373–375. 13. Salhi B, Stettner E. In defense of the use of lidocaine in rapid sequence intubation. Ann Emerg Med. 2007;49(1):84-86.

8


Critical Decisions in Emergency Medicine / Neurologic Edition

14. Caro DA, Bush S. Pretreatment agents. In: Walls RM, Murphy MF, Luten RC, eds. Manual of Emergency Airway Management. 3rd ed. Philadelphia, PA: Lippincott Williams & Wilkins; 2008:222-232. 15. Roberts IG, Schierhout G. Hyperventilation therapy for acute traumatic brain injury. Cochrane Database of Systematic Reviews. 1997, Issue 4. Art. No.: CD000566. DOI: 10.1002/14651858.CD000566. 16. Brain Trauma Foundation, American Association of Neurological Surgeons, Congress of Neurological Surgeons, et al. Guidelines for the management of severe traumatic brain injury. XIV. Hyperventilation. J Neurotrauma. 2007;24(1):S87-S90. 17. Schwartz DT. Hydrocephalus—use of CT before LP. In: Emergency Radiology: Case Studies. New York, NY: McGraw-Hill; 2007:487-497.

Additional Readings Lueck C, McIllwaine G. Interventions for idiopathic intracranial hypertension. Cochrane Database of Systematic Reviews. 2005, Issue 3. Rangwala L, Liu G. Pediatric idiopathic intracranial hypertension. Surv Ophthalmol. 2007;52:597-612. Shah SM, Kelly KM, eds. Increased intracranial pressure, Chapter 26; Normal pressure hydrocephalus, Chapter 28; Hydrocephalus and shunts in children, Chapter 34. Emergency Neurology Principles and Practice. New York, NY: Cambridge University Press; 1999:378-380. Shah VA, Kardon RH, Lee AG, et al. Long-term follow-up of idiopathic intracranial hypertension: the Iowa experience. Neurology. 2008;70:634-640. Tabassi K, Silvestre CG. Management of increased intracranial pressure and intracranial shunts. In: Roberts JR, Hedges JR, eds: Clinical Procedures in Emergency Medicine. 4th ed. Philadelphia, PA: Elsevier Science; 2004:1185-1196.

Acknowledgment The authors would like to thank Dr. H. Edward Seibert and Dr. Timberly Booker for the use of case three. It informed this lesson greatly on the significant morbidity and mortality that can arise with increased intracranial pressure.

9


Critical Decisions in Emergency Medicine / Neurologic Edition

1. Which of the following findings is most suggestive of increased intracranial pressure (ICP) in an infant? A. Bulging or tense fontanelles B. Drowsiness C. Failure to thrive D. Headache 2. For patients with suspected acutely increased ICP, which of the following is the first-line imaging study? A. Brain CT with contrast B. Brain CT without contrast C. Magnetic resonance angiography of the brain D. Magnetic resonance imaging of the brain with and without gadolinium 3. What is the leading cause of acute hydrocephalus? A. Infections such as bacterial meningitis B. Ischemic stroke C. Neurocysticercosis D. Subarachnoid hemorrhage 4. Elevated cerebrospinal fluid (CSF) opening pressure is defined as a value greater than: A. 5 cm H2O B. 10 cm H2O C. 15 cm H2O D. 25 cm H2O in an obese patient 5. Treatment of idiopathic intracranial hemorrhage is aimed at preventing which of the following? A. Herniation B. Optic nerve atrophy C. Retinal hemorrhage D. Stroke

6. Which of the following is the first-line drug treatment for lowering ICP for stable patients with idiopathic intracranial hypertension? A. Acetazolamide B. Furosemide C. Glucagon D. Mannitol 7. Which of the following is no longer routinely recommended during induction in patients with elevated ICP? A. Atropine B. Etomidate C. Fentanyl D. Lidocaine 8. Which of the following induction agents has been associated with a transient increase in ICP? A. Etomidate B. Fentanyl C. Propofol D. Succinylcholine 9. Which of the following interventions should be delayed 24 hours after intubation in a patient with increased ICP for whom the intervention would otherwise be indicated? A. Hypertonic saline B. Hyperventilation C. Mannitol D. Rapid sequence intubation 10. It is relatively safe to perform a lumbar puncture in a patient with suspected meningitis who has which of the following findings? A. Communicating hydrocephalus B. Compression of basal cisterns C. Midline shift D. Obstructive hydrocephalus

10


Critical Decisions in Emergency Medicine / Neurologic Edition

Evaluation and Treatment of Status Epilepticus Lesson 2

Julie J. Cooper, MD, and Michael A. Gisondi, MD, FACEP

â– Objectives On completion of this lesson, you should be able to: Describe presentations of status epilepticus in both adult and pediatric populations. 1. Discuss appropriate first- and second-line medications for seizure termination in status epilepticus. 2. Select appropriate medications for rapid sequence intubation (RSI) and postintubation infusion therapy in status epilepticus patients requiring airway management. 3. Describe an appropriate diagnostic evaluation for a patient presenting with status epilepticus. 4. Explain how to differentiate simple from complex febrile seizures in children and describe effective treatments for children with febrile status epilepticus.

â– From the EM Model 12.0 Nervous System Disorders 12.9 Seizure Disorders

Seizures affect more than three million Americans of all ages at an estimated $15.5 billion in direct and indirect health care costs.1 Seizurerelated emergencies are a common reason for emergency department visits; approximately 1.2% of all emergency department patients present with complaints related to seizures.2 The annual incidence of status epilepticus in adults is estimated to be between 18 and 40 per 100,000 in the United States,3 and in children the incidence is about 31,600 per year.4 Status epilepticus is a medical emergency responsible for up to 42,000 deaths annually in the United States; it occurs in a wide spectrum of patients including those with no prior history of seizure.5 The traditional definition of status epilepticus is continuous seizure activity lasting 30 minutes or a succession of seizures without an intervening return to baseline mental status. This definition uses 30 minutes as a threshold duration based mainly on animal data demonstrating that neurons undergo irreversible damage after 30 minutes of continually induced seizures.6 This definition is problematic in the clinical setting; however, because the type of seizure and the time to neuronal damage will vary from patient to patient. There are no readily available laboratory or imaging tests to quantify this ongoing brain damage. Some experts suggest that 5 to 10 minutes of continuous seizure activity is a more practical definition

of status epilepticus, recognizing that more than 90% of tonic-clonic seizures will spontaneously terminate within 2 minutes.3 Pediatric data show that once a seizure has lasted for 5 to 10 minutes it is unlikely to terminate without pharmacologic intervention, and the longer a seizure lasts the more difficult it is to control despite medical therapy.7 Operationally, status epilepticus must be presumed in any patient who presents to the emergency department with persistent seizure activity, because most prehospital transport times will easily exceed 5 minutes.

Case Presentations â– Case One A 67-year-old man is brought to the emergency department by ambulance after having a generalized tonic-clonic seizure. According to his wife, the patient was in his usual state of health until this morning. He was in the kitchen when she heard a loud noise and found him on the floor having a generalized seizure. When EMS arrived, the seizure had stopped, and the patient was noted to appear postictal, with a depressed mental status. There was no obvious trauma. He has a history of hypertension, hyperlipidemia, and prior prostatectomy for prostate cancer. The paramedics were not able to establish intravenous access in the ambulance, and only 100% oxygen by face mask was given as he was transported to the emergency department. Shortly after being placed in an examination 11


Critical Decisions in Emergency Medicine / Neurologic Edition

Critical Decisions • Which medications should be used in the emergency department management of status epilepticus?

• What diagnostic testing in the emergency department can help reveal the causes of status epilepticus?

• What additional therapies may be used to treat refractory status epilepticus?

• What are the unique considerations in pediatric status epilepticus?

• What are the special considerations in airway management for status epilepticus? room, he begins to have tonic-clonic seizure activity. Initial vital signs are blood pressure 140/80, pulse rate 110, respiratory rate 12, temperature 37.3°C (99.1°F), and oxygen saturation of 99% on 100% oxygen via face mask. Physical examination reveals a well-developed, well-nourished elderly man having a tonic-clonic seizure. He is unresponsive to voice or painful stimulus. A cervical collar is in place, and no trauma is noted to the head or face. Pupils are equal, round, and reactive to light. There is scant bright red blood noted in the oropharynx. Heart, lung, and abdominal examinations are unremarkable. Peripheral intravenous access is obtained, and an initial point-of-care blood glucose level is 143. ■ Case Two An 18-month-old otherwise healthy girl is brought to the emergency department after a seizure witnessed by her babysitter. She had upper respiratory symptoms over the past 2 days and had two episodes of nonbloody, nonbilious emesis after breakfast. This morning she was interactive and playful. She lay down for a nap and about 45 minutes later was noted to have a generalized seizure of at least 5 minutes’ duration. EMS was called and found a lethargic toddler who aroused to voice and was irritable. In the ambulance she slowly returned to her baseline mental status. The babysitter states that the child has no known prior history of seizure. Initial vital signs are blood pressure 90/palpable, pulse rate 130, respiratory rate 12, temperature 39.7°C (103.5°F), and oxygen

saturation 99% on room air. Physical examination reveals an alert toddler who is irritable and cries when examined. No trauma is noted. Pupils are equal, round, and reactive to light. Mucous membranes are moist. The oropharynx is clear. No cervical lymphadenopathy is noted. She has a normal heart, lung, and abdominal examination with no organomegaly. A peripheral intravenous line is established. About 10 minutes after arriving in the emergency department, the child experiences another generalized seizure.

Classification of Status Epilepticus Status epilepticus can be classified as convulsive or nonconvulsive. Generalized tonic-clonic status epilepticus is the most common and serious form and might represent cortical generalization of a precipitating partial seizure. Other convulsive forms are simple partial seizures (focal motor seizures, focal sensory symptoms, or cognitive symptoms without impaired consciousness) and complex partial seizures (with impaired consciousness). Nonconvulsive status epilepticus can present a diagnostic challenge because its manifestations are subtle and can include absence status and myoclonic status (repeated myoclonic jerks in the setting of altered mental status). Psychogenic status epilepticus (or psychogenic nonepileptic status epilepticus, PNESE) presents with generalized motor features of seizure without true epileptiform activity on electroencephalograph (EEG).

Features that can help distinguish psychogenic status epilepticus include gradual onset of undulating motor activity, variable motor activity, or partially preserved consciousness with a lack of a postictal period. Response to noxious stimuli in these patients is often preserved even during “seizure” activity.8

Assessment and Management of Status Epilepticus Seizure patients can present to an emergency department in dramatic fashion and require a rapid and organized diagnostic assessment in parallel with early and aggressive anticonvulsant therapy.9 Status epilepticus is a medical emergency and requires mobilization of resources and rapid diagnostic assessments similar to those required for cardiac arrest or an ST-elevation myocardial infarction. As with any life-threatening illness, the standard considerations of airway, breathing, and circulation apply in status epilepticus. Endotracheal intubation is not routinely indicated solely for seizure activity but is frequently necessitated during anticonvulsant infusion therapy. Place all actively seizing patients on supplemental oxygen by face mask and monitor oxygen saturation continuously via pulse oximetry. If feasible, establishing a nasopharyngeal airway could facilitate improved oxygenation.3 Initial vital signs and intravenous access should be obtained with the goal of rapid administration of a benzodiazepine in conjunction with the ongoing 12


Critical Decisions in Emergency Medicine / Neurologic Edition

diagnostic evaluation. Place the patient on continuous telemetry monitoring and obtain an ECG when possible. Obtain a point-of-care blood glucose level. If dextrose is indicated, consider pretreatment with 100 mg of intravenous thiamine, especially if there is concern for alcoholism or malnutrition. CRITICAL DECISION Which medications should be used in the emergency department management of status epilepticus?

There are four categories of anticonvulsant drugs used to treat status epilepticus, and they are generally initiated and titrated rapidly in a stepwise fashion. These categories are the benzodiazepines, phenytoin, barbiturates, and propofol. Levetiracetam and valproate have recently been added to this list as alternatives to phenytoin, given emerging data for their effectiveness in status epilepticus. Table 1 summarizes these medications, their dosages, and considerations for their use. First-Line Therapy: Benzodiazepines Benzodiazepines remain the first-line treatment for all seizure emergencies and status epilepticus. Their mechanism of action is enhancement of chloride conduction through central nervous system (CNS) Îł-aminobutyric acid type A (GABA A) receptors, decreasing overall neuronal excitability.10 In the largest pediatric clinical trial to date, lorazepam alone (0.1 mg/kg) terminated seizure activity in 64.9% of patients with overt convulsive activity.11 A 2005 Cochrane review12 of anticonvulsant therapy also found parenteral lorazepam to be the initial therapy of choice in adults presenting with status epilepticus. Additionally, benzodiazepines have been shown to be safe when given in the prehospital setting with the goal of aborting seizure activity prior to emergency department arrival. Of note, patients who were still in status epilepticus at the time of emergency department

arrival were more than twice as likely to require ICU admission as those whose seizures were successfully terminated prehospital.13 Lorazepam is familiar to emergency department providers, fast acting, easy to use, and offers a longer therapeutic effect and lower risk of respiratory suppression than diazepam. There is no upper limit to the amount of benzodiazepine that can be administered. Dosing may be limited by respiratory suppression in some settings, but in the emergency department the physician should secure the airway and continue repeating benzodiazepine administration until seizure termination; cessation of seizure activity should supercede the concern for respiratory depression. Second-Line Therapy: Phenytoin, Fosphenytoin, Levetiracetam, Valproate Phenytoin acts to terminate seizure activity through modulation of neuronal voltage-dependent sodium and calcium channels. Phenytoin is a familiar anticonvulsant drug in the emergency department that is widely used as a second-line agent when seizures are not controlled with benzodiazepines, although this strategy has yet to be rigorously clinically tested.3 Phenytoin has some well-recognized limitations as a parenteral therapy; side effects include hypotension, cardiac arrhythmias (similar in mechanism to class IA antiarrhythmic drugs), pain at the infusion site, and local toxicity if it extravasates (purple glove syndrome). All of the systemic toxicities are infusion-rate related and largely due to the propylene glycol used as a diluent. Given the cardiovascular side effects, patients should be on a telemetry monitor, and phenytoin should be infused at a rate to limit side effects, generally no faster than 50 mg/min. Phenytoin cannot be administered by intravenous push. It is often ineffective in status epilepticus caused by alcohol withdrawal or toxicologic etiologies.

Fosphenytoin is a prodrug of phenytoin that is not diluted in propylene glycol; thus it causes fewer adverse reactions and can be infused much faster. Fosphenytoin must undergo first-pass metabolism in the liver, where it is converted to phenytoin; therefore its overall time to onset of action is similar to that of slowly infused phenytoin. It is frequently cited as a less desirable parenteral therapy for seizures in the emergency department because of its higher cost; however, the more favorable side-effect profile of fosphenytoin and its ease of rapid infusion can make it a better choice.14 Levetiracetam is an anticonvulsant drug with a poorly defined mechanism of action that has become widely used in the outpatient setting for the treatment of seizure disorders because of its efficacy and favorable side-effect profile. A parenteral formulation was approved in 2006. Although animal studies of intravenous levetiracetam for status epilepticus have been equivocal, there are case reports of refractory status epilepticus that has responded to levetiracetam in the ICU setting. There are no studies of intravenous levetiracetam in the emergency department setting, but as it becomes more widely used it may have a role as a lower-toxicity second-line therapy.15 Three small trials of intravenous sodium valproate have demonstrated its efficacy in refractory status epilepticus. Studies have shown a benefit of valproate over phenytoin as a first-line anticonvulsant drug for status epilepticus,16 equivalence of valproate and phenytoin in patients who did not respond to first-line diazepam (approximately 85% of these patients had termination of seizure activity with a second-line agent),17 and similar response of pediatric status epilepticus to both diazepam infusion and valproate as first-line therapy (with a reduction in respiratory depression in children who received valproate).18 Some authors advocate that levetiracetam or 13


Critical Decisions in Emergency Medicine / Neurologic Edition

valproate be recommended as thirdline therapies before barbiturates.3 Refractory Status Epilepticus A critical juncture in the management of status epilepticus occurs when a patient continues to seize after an adequate dose

of benzodiazepine has been administered. Second-line anticonvulsant drugs often take some time to be prepared and have an onset of action of 10 to 30 minutes. In one study, only 7% of patients who did not respond to appropriate doses of a benzodiazepine and phenytoin or

fosphenytoin responded to any thirdline agent.10 Barbiturates are frequently cited as an appropriate third-line agent in the ICU setting. Their mechanism of action is similar to benzodiazepines in that they enhance CNS GABA activity and decrease neuronal excitability.

Table 1. Medications Commonly Used in the Emergency Department Management of Status Epilepticus Benzodiazepines – First-Line Therapy Medication Routes of Administration, Dosage Lorazepam IV or IM Adults: 0.02-0.03 mg/kg IV every 5 min, maximum single dose 4 mg Children: 0.1 mg/kg IV, maximum single dose 4 mg Diazepam IV, IM, rectal Adults: 0.05-0.1 mg/kg IV every 5 min; Children: 0.3-0.5 mg/kg IV or PR, maximum, 10 mg Midazolam IV, IM, rectal, buccal, intranasal, or continuous infusion Infusion: 0.05 mg/kg; Adults: 0.1-0.2 mg/kg IV; Buccal in children: 0.5 mg/kg Anticonvulsant Drugs – Second-Line Therapy Medication Routes of Administration, Dosage Phenytoin IV only for status epilepticus 20 mg/kg loading dose; goal concentration 15-25 mcg/mL Infuse at a rate of up to 50 mg/min, titrate down if cardiovascular toxicity; give at 25 mg/min if patient is older than 50 years or ectopy is noted on monitor Fosphenytoin IV or IM Prescribed as “phenytoin equivalent” dose Sodium Valproate

IV or rectal 20-30 mg/kg loading dose at a rate of up to 10 mg/ kg/min

Levetiracetam

No established IV dose for use in status epilepticus, small studies used doses of 1,000-6,000 mg; may also be given orally through a nasogastric tube

Barbiturates Medication Phenobarbital Pentobarbital

Infusion Therapy Medication Propofol

Considerations First-line agent for status epilepticus

Can be given as a rectal gel when prompt intravenous access is not possible Fast effect (3 min IV, 5-7 min rectal) The benzodiazepine of choice in hemodynamically unstable or ICU patients; used in infusion therapy

Considerations Traditional second-line anticonvulsant drug after the benzodiazepines; can cause hypotension, drug rashes, irritation on infusion, purple-hand syndrome if it extravasates; draw serum level about 10 min after infusion ends and administer an extra 5 mg/kg if serum levels are subtherapeutic Given more rapidly, less likely to cause hypotension, more expensive than phenytoin, IM absorption can be erratic Frequently used when phenytoin/fosphenytoin are contraindicated; adverse effects include hyperammonemia, pancreatitis, thrombocytopenia (contraindicated in preexisting thrombocytopenia) No human controlled trials of its use in status epilepticus

Routes of Administration, Dosage IV or IM 20 mg/kg loading dose at a rate of 30-50 mg/min IV 10 mg/kg of pentobarbital infused at a rate up to 100 mg/min; may follow with continuous infusion

Considerations Many patients need intubation after loading dose; long half-life with slow emergence Mandatory central venous pressure monitoring makes barbiturate coma appropriate only in the ICU setting

Routes of Administration, Dosage IV bolus then infusion (off-label) 1-2 mg/kg bolus followed by 1.5-10 mg/kg/hr infusion

Considerations Excellent postintubation infusion therapy; can be titrated to effect (cessation of seizure clinically or, if paralyzed, by EEG)

14


Critical Decisions in Emergency Medicine / Neurologic Edition

Phenobarbital, thiopental, and its metabolite pentobarbital are of limited utility in the emergency department because of their slow onset of action and because they induce profound hypotension requiring continuous central venous pressure monitoring.6 CRITICAL DECISION What additional therapies may be used to treat refractory status epilepticus?

Given the low rate of success of third-line agents, many clinicians now prefer proceeding directly to continuous infusion therapy rather than using a third-line anticonvulsant drug.3 Continuous infusions used in the management of status epilepticus include midazolam, which is preferred in hemodynamically unstable patients for its ease of titration and short half-life,3 and propofol, an intravenous anesthetic agent that is familiar to most emergency department providers and has very effective anticonvulsant properties. Propofol is used off-label as an infusion therapy for status epilepticus. CRITICAL DECISION What are the special considerations in airway management for status epilepticus?

Most of the medications used in the management of status epilepticus have the potential to cause respiratory depression. Adequate dosing of anticonvulsant drugs should never be withheld for a competing goal of avoiding intubation, as refractory seizures must be stopped as quickly as possible to limit neuronal damage. Most patients with status epilepticus that require intubation will not yet have a clear etiology of their seizures, so RSI should be performed with consideration for potential intracranial pathology. Consider pretreatment with lidocaine if there is a concern for increased intracranial pressure and time allows. Most of the anticonvulsant drugs cause some degree of hypotension and sedation; if an additional induction agent is required for intubation,

consider using etomidate as it has a favorable hemodynamic profile. Use a nondepolarizing paralytic agent such as rocuronium to avoid increases in intracranial pressure. Paralyzing status epilepticus patients will convert convulsive status epilepticus to an iatrogenic nonconvulsive status epilepticus, as motor activity is blunted while neuronal stimulation is on-going. Infusion therapy should begin immediately after intubation either with midazolam or propofol to treat presumed nonconvulsive status epilepticus. Continuous EEG monitoring should be arranged as quickly as possible in order to titrate infusion therapy to seizure termination. CRITICAL DECISION What diagnostic testing in the emergency department can help reveal the causes of status epilepticus?

Ongoing management of status epilepticus requires a search for correctable underlying causes of seizure activity. In adults, the most common factors leading to status epilepticus are subtherapeutic anticonvulsant drug levels (34%), history of prior CNS insult (25%), cerebrovascular accident (22%), metabolic derangements (15%), and alcohol-related seizures (13%).5 In pediatric populations, the most common etiologies are infection outside the CNS (febrile status epilepticus, 52%), prior CNS disease (most commonly congenital malformations, 38%), and subtherapeutic anticonvulsant drug levels in children with known epilepsy (21%).5 The diagnostic evaluation for the etiology of status epilepticus should begin in parallel with anticonvulsant therapy. There are few rigorously studied guidelines for the diagnostic evaluation of status epilepticus, and most recommendations represent expert opinion. Routine blood testing, including bedside glucose, electrolyte levels, creatine

phosphokinase to evaluate ongoing muscle breakdown, and a toxicology screen, should be performed in all adult patients presenting with status epilepticus.3,6,19 Serum anticonvulsant drug levels should be drawn if the patient’s anticonvulsant medication regimen is known. Although status epilepticus seldom has a surgically correctable intracranial etiology,5 computed tomography (CT) scanning of the head remains an important screening tool in cases of suspected trauma and for patients who cannot undergo a complete neurologic examination because of depressed or altered mental status and could go on to require lumbar puncture. Head CT should not delay initiation of anticonvulsant drug therapy. Lumbar puncture is indicated in patients with preceding illness, fever, or other signs of CNS infection but is not routinely indicated in status epilepticus. Consider blood cultures in the febrile seizure patient only; the diagnostic yield of blood cultures in an afebrile patient is unclear.4 CRITICAL DECISION What are the unique considerations in pediatric status epilepticus?

Febrile status epilepticus is the cause of approximately 22% of childhood status epilepticus.4 Febrile seizures are the most common form of seizures, affecting 2% to 5% of all children and usually appearing when children are between 3 months and 5 years of age. Febrile seizures are classified as simple or complex. A simple febrile seizure lasts less than 15 minutes and is self-limited, with tonic-clonic features. Children with simple febrile seizures have no recurrence within 24 hours and no postictal pathology. In contrast, a complex febrile seizure is of longer duration (>15 minutes), or it can present as a series of shorter seizures recurring within 24 hours. Focal febrile seizures (such as partial seizures with secondary generalization or deviation of the head or eyes) are considered complex febrile seizures.20

15


Critical Decisions in Emergency Medicine / Neurologic Edition

Although only supportive care is routinely required for simple febrile seizures, febrile status epilepticus is treated using the same medication algorithm as outlined for adults. Benzodiazepines are highly effective as first-line therapy in febrile status epilepticus, with rare cases necessitating the addition of secondline anticonvulsant drugs. Despite the association between seizure and fever, antipyretic medications have not been shown to be beneficial in febrile seizures or status epilepticus.20 Status epilepticus, whether febrile or afebrile, is the first seizure presentation in 12% of children with epilepsy.21 Unique to the pediatric population is the widespread use of rectal and buccal benzodiazepines when intravenous access is difficult to obtain.22 Prehospital benzodiazepine use for children with status epilepticus has been shown to be associated with shorter seizure duration compared to those whose treatment was delayed until emergency department arrival.13

Case Resolutions ■ Case One This 80-kg, 67-year-old man received an initial dose of 4 mg lorazepam intravenously (standard dose 0.02-0.03 mg/kg IV in adults). He continued to have tonic-clonic seizure activity, and a second dose of lorazepam was administered 5 minutes later; fosphenytoin was ordered from the pharmacy. After a third dose of lorazepam, his respiratory rate decreased to 8 with intermittent oxygen desaturations to 88% on 100% oxygen via face mask. The decision was made to intubate for airway protection and to facilitate diagnostic workup. Rocuronium, 80 mg, was administered, and the patient was endotracheally intubated on the first attempt. Neurology was consulted for continuous EEG monitoring and ICU admission. His seizure activity on EEG terminated with initiation of propofol infusion. Magnetic resonance imaging later revealed a left middle cerebral artery

distribution cerebrovascular accident. ■ Case Two The emergency physician recognized the events as a complex febrile seizure in this toddler. During the second seizure, a dose of 0.1 mg/ kg of lorazepam was administered intravenously with prompt cessation of seizure activity. Empiric antibiotics for meningitis were ordered, and a lumbar puncture revealed a pleocytosis. No further seizures were observed in the emergency department. The child was admitted to the pediatric ICU for observation and antibiotic therapy.

Summary Status epilepticus is a timesensitive medical emergency that can present in a dramatic fashion.

Emergency department management requires rapid recognition and early administration of appropriate anticonvulsant drug therapy to help prevent neuronal damage and longterm sequelae. The longer a seizure continues, the more difficult it is to control. Most status epilepticus seizures will terminate with early administration of a benzodiazepine and phenytoin or fosphenytoin, but emergency physicians should be prepared to treat refractory cases after endotracheal intubation with infusion therapy using midazolam or propofol. Treatment of status epilepticus should occur in parallel with an appropriate diagnostic evaluation, early neurology service consultation, and ICU admission.

Pearls • Any patient presenting to an emergency department with an active seizure should be considered to be in status epilepticus, because most hospital transport times exceed the 5-minute operational definition of status epilepticus. • Simple febrile seizures in children should be treated supportively, but complex febrile seizures and febrile status epilepticus should be treated with anticonvulsant drugs. • Consider nonconvulsive status epilepticus in the differential diagnosis of patients presenting with altered mental status or reduced responsiveness. • Potential systemic complications of status epilepticus include rhabdomyolysis, aspiration, disseminated intravascular coagulation, neurocardiogenic pulmonary edema, arrhythmia, cardiac arrest, endocrinopathies, electrolyte disturbances, and cerebral edema. These complications should be actively sought out and aggressively treated in the emergency department.

Pitfalls • Withholding adequate doses of anticonvulsant drugs out of concern for respiratory depression. “Time is brain,” and stopping the seizure is the priority; intubate for airway protection and ventilation. • In cases of refractory seizures, failing to search for uncommon, but reversible causes of status epilepticus such as hypoglycemia, isoniazid toxicity, hyponatremia, and postpartum eclampsia. • Failing to remember that dosing of phenytoin is weight-based—18 to 20 mg/kg (or 1,400 mg in a 70-kg adult); it is easy to underdose patients when not taking their weight into consideration. Infuse phenytoin at a rate of no more than 50 mg/minute in order to avoid cardiovascular side effects. Never give phenytoin as an intravenous push. If ectopy is observed, halve the infusion rate.

16


Critical Decisions in Emergency Medicine / Neurologic Edition

References 1. Epilepsy foundation. Epilepsy and seizure statistics. Available online at: http://www.epilepsyfoundation. org/about/statistics.cfm. Accessed June 8, 2009. 2. Huff JS, Morris DL, Kothari RU, et al, Emergency Medicine Seizure Study Group. Emergency department management of patients with seizures: a multicenter study. Acad Emerg Med. 2001;8:622-628. 3. Costello DJ, Cole AJ. Treatment of acute seizures and status epilepticus. J Intensive Care Med. 2007;22(6):319-347. 4. Riviello JJ, Ashwal S, Hirtz D, et al. Practice parameter: diagnostic assessment of the child with status epilepticus (an evidence-based review): report of the Quality Standards Subcommittee of the American Academy of Neurology and the Practice Committee of the Child Neurology Society. Neurology. 2006;67:1542-1550. 5. DeLorenzo RJ, Pellock JM, Towne AR, et al. Epidemiology of status epilepticus. J Clin Neurophysiol. 1995;12(4):316-325. 6. García Peñas JJ, Molins A, Salas Puig J. Status epilepticus: evidence and controversy. Neurologist. 2007;13(6 Suppl 1):S62-S73. 7. Shinnar S, Berg AT, Moshe SL, et al. How long do new-onset seizures in children last? Ann Neurol. 2001;49:659-664. 8. Reuber M, Baker GA, Gill R, et al. Failure to recognize psychogenic nonepileptic seizures may cause death. Neurology. 2004;62:834-835. 9. Tarabar AF, Ulrich AS, D’Onofrio G. Seizures. In: Adams JG, Barton ED, Collings JL, et al, eds. Emergency Medicine. Philadelphia, PA: Saunders Elsevier; 2008. 10. Treiman DM, Meyers PD, Walton NY, et al. A comparison of four treatments for generalized convulsive status epilepticus. Veterans Affairs Status Epilepticus Cooperative Study Group. N Engl J Med. 1998;339:792-798. 11. Treiman DM, Meyers PD, Walton NY, et al. A comparison of four treatments for generalized convulsive status epilepticus. New Engl J Med. 1998;339:792-798. 12. Prasad K, Al-Roomi K, Krishnan PR, Sequeira R. Anticonvulsant therapy for status epilepticus. Cochrane Database of Systematic Reviews 2005. Issue 4. CD003723. 13. Alldredge BK, Gelb AM, Isaacs SM, et al. A comparison of lorazepam, diazepam, and placebo for the treatment of out-of-hospital status epilepticus. N Engl J Med. 2001;345:631-637. 14. Rudis MI, Touchette DR, Swadron SP, et al. Costeffectiveness of oral phenytoin, intravenous phenytoin and intravenous fosphenytoin in the emergency department. Ann Emerg Med. 2004;43(3):386-397. 15. Misra UK, Kalita J, Patel R. Sodium valproate vs. phenytoin in status epilepticus: a pilot study. Neurology. 2006;67:340–342. 16. Agarwal P, Kumar N, Chandra R, et al. Randomized study of intravenous valproate and phenytoin in status epilepticus. Seizure. 2007;16:527–532. 17. Mehta V, Singhi P, Singhi S. Intravenous sodium valproate versus diazepam infusion for the control of refractory status epilepticus in children: a randomized controlled trial. J Child Neurol. 2007;22:1191–1197. 18. Wheeles JW, Traiman DM. The role of newer antiepileptic drugs in the treatment of generalized convulsive status epilepticus. Epilepsia. 2008;49(suppl 9):74-78. 19. 19. Pellock JM. Overview: definitions and classifications of seizure emergencies. J Child Neurol. 2007;22(5):9s-13s. 20. Fetveit A. Assessment of febrile seizures in children. Eur J Pediatr. 2008;167:17-27. 21. Haut SR, Shinnar S, Moshe SL, et al. The association between seizure clustering and convulsive status epilepticus in patients with intractable complex partial seizures. Epilepsia. 1999;40:1832-1834. 22. McIntyre J, Robertson S, Appleton R, et al. Safety and efficacy of buccal midazolam versus rectal diazepam for emergency treatment of seizures in children: a randomized controlled trial. Lancet. 2005;366:205-210.

17


Critical Decisions in Emergency Medicine / Neurologic Edition

11. A man arrives by ambulance having a generalized tonicclonic seizure. No past history is available, and the bystanders who called EMS are not present. His airway is intact, and his vital signs are stable. What is the most appropriate initial action? A. 18-mg/kg loading dose of phenytoin B. Airway control with rapid sequence intubation (RSI) C. Immediate administration of 0.02 mg/kg of lorazepam D. Neurologic consultation

17. Which of the following statements is true regarding the use of benzodiazepines in status epilepticus? A. Benzodiazepines are less effective than phenytoin or fosphenytoin for seizures from toxicologic causes B. Menzodiazepines should be titrated to respiratory depression C. Most patients have termination of seizure activity with a benzodiazepine alone D. Most patients require endotracheal intubation after the first dose of benzodiazepine

12. A 60-year-old man presents in status epilepticus. During treatment he develops hypotension refractory to intravenous fluid; QT prolongation with frequent premature ventricular contractions is noted. Which of the following most likely caused these findings? A. Fosphenytoin B. Levetiracetam C. Lorazepam D. Phenytoin

18. Which of the following anticonvulsant medications is contraindicated for a patient with idiopathic thrombocytopenic purpura? A. Levetiracetam B. Lorazepam C. Phenytoin D. Sodium valproate

13. A 65-year-old man with refractory status epilepticus is intubated for respiratory depression. He is still actively seizing after receiving three doses of lorazepam and initiation of a fosphenytoin load. RSI is performed using lidocaine, etomidate, and rocuronium. A continuous EEG shows ongoing seizure activity. Which is the most appropriate next action? A. Continue to push 0.1-mg/kg doses of lorazepam every 5 minutes until seizure cessation B. Initiate a propofol infusion and titrate upward until cessation of seizure on EEG monitor C. Initiate a rocuronium infusion to continue to suppress convulsions D. Repeat the fosphenytoin load via an intramuscular route 14. What is the most common cause of status epilepticus in adults with a history of epilepsy? A. Alcohol withdrawal B. Central nervous system infection C. Cerebrovascular accident D. Subtherapeutic anticonvulsant drug level

19. A man is brought in with status epilepticus. He appears to be about 40 years old. His seizures are refractory to multiple doses of intravenous lorazepam and fosphenytoin. After RSI and initiation of propofol infusion therapy, a chest radiograph is obtained that shows a right upper lobe cavitary infiltrate. His glucose is normal, and he has a severe metabolic acidosis. What is the most appropriate empiric treatment for this 80-kg adult? A. Enoxaparin, 80 mg subcutaneously B. Magnesium sulfate, 6 g IV C. Piperacillin/tazobactam, 4.5 g IV, and vancomycin, 1 g IV D. Pyridoxine, 5 g IV 20. Which of the following is true regarding fosphenytoin? A. The adverse effect profile for fosphenytoin is similar to that for intravenous phenytoin B. The cost per dose for fosphenytoin is about the same as for intravenous phenytoin C. The diluent for fosphenytoin is similar to that for phenytoin D. Intravenous fosphenytoin can safely be administered faster than intravenous phenytoin

15. In which of the following children are supportive care and observation appropriate emergency department management strategies? A. 12-day-old infant with fever to 38.3째C (101째F) and a generalized seizure lasting 4 minutes followed by an apneic spell, who now looks well B. 9-month-old baby boy with known otitis media on amoxicillin, fever to 38.3째C (101째F), and 4-minute tonic-clonic seizure that terminated spontaneously at home; he now looks well C. 14-month-old toddler with three febrile seizures today, the longest of which lasted 3 minutes, who appears septic D. 5-year-old girl with fever, lethargy, and witnessed seizure involving head and eye deviation to the left for 10 minutes, now postictal 16. Which of the following RSI drugs should theoretically be avoided when intubating an undifferentiated patient with status epilepticus? A. Etomidate B. Lidocaine C. Midazolam D. Succinylcholine

18


Critical Decisions in Emergency Medicine / Neurologic Edition

Peripheral Neuropathies Lesson 3

David Della-Giustina, MD, FACEP, FAWM, and Tristan Knutson, MD

■ Objectives On completion of this lesson, you should be able to: 1. Discuss the different types of botulism. 2. Describe the clinical presentation and diagnostic strategies for botulism. 3. Explain the treatment modalities for each type of botulism. 4. Describe common clinical findings in Guillain-Barré syndrome. 5. Describe the pitfalls of relying on laboratory testing in attempting to diagnose Guillain-Barré syndrome. 6. Explain the proper disposition of a patient diagnosed in the emergency department with suspected GuillainBarré syndrome. 7. Describe the physical examination findings in patients with carpal tunnel syndrome.

■ From the EM Model 12.0 Nervous System Disorders 12.7 Neuromuscular Disorders

The evaluation and management of patients who present with peripheral neurologic complaints can cause anxiety and frustration for emergency physicians because of the wide variety of benign and life-threatening illnesses that are associated with those complaints and the fact that patients can present early in the disease process when it is more difficult to discern a specific diagnosis. Additionally, patients with peripheral neuropathies are generally only a small percentage of the patients that a typical emergency physician sees. In this review, one common and two uncommon yet life-threatening peripheral neuropathies will be discussed. The focus is to provide emergency physicians with expertise in making the diagnosis in those early and uncertain cases. Common pitfalls in the management of these peripheral neuropathies will also be reviewed. In order to diagnose a peripheral neuropathy, it is first important to differentiate it from a central neuropathy, which can be difficult, because features of the history and physical examination for both overlap. Important elements to elicit in the history are the symptom onset, duration, provocative and alleviating factors, prodromal symptoms, and the degree of the neurologic deficit. The neurologic examination should focus on the following areas: sensory, motor, reflexes, and upper motor neuron signs. On the sensory examination, peripheral lesions are generally

restricted to a single dermatome with a sharp border of demarcation. With central lesions, especially spinal lesions, sensory symptoms are more commonly bilateral and involve multiple dermatomes. One area that is commonly overlooked on the sensory examination is the trunk. Specifically, patients with a thoracic spine lesion may have sensation that is symmetric and present over the legs, but in reality and in comparison to the trunk it is diminished but just not recognized by the patient or examiner. The classic stockingglove sensory deficit is a peripheral neuropathy that is most commonly seen with diabetes and chronic alcohol use. On motor examination, peripheral lesions usually cause unilateral weakness and are limited to the muscle group that is innervated by the involved nerve. The major exception to this finding is GuillainBarré syndrome (GBS) because it is a peripheral polyneuropathy. A central neuropathy will have weakness or paralysis distal to the site of the lesion. The findings with these central lesions are more commonly bilateral and can have associated upper motor neuron signs such as spasticity, hyperreflexia, and clonus. In some instances, the patient will complain of leg weakness but have no demonstrable weakness on the initial examination. One maneuver to help demonstrate weakness is to have the patient squat down and rise to the standing position without any assistance. Most patients without 19


Critical Decisions in Emergency Medicine / Neurologic Edition

Critical Decisions • What clinical clues suggest the diagnosis of botulism? • Are there any confirmatory tests for botulism that can be performed in the emergency department?

• A patient who has motor weakness also has sensory deficits and complains of pain; does this rule out Guillain-Barré syndrome (GBS)? • Should all patients with suspected GBS undergo a lumbar puncture in the emergency department?

• What immediate life threats must be addressed in a patient with botulism? • Other than supportive care, what can be done for a patient with botulism?

• Do all patients with suspected GBS require hospital admission, and if so should it be to the ICU? • What is the most effective, nonsurgical treatment modality for carpal tunnel syndrome?

weakness are able to do this with only minor difficulty. If the patient is unable to rise or cannot rise without assistance then they have weakness. A second maneuver to differentiate weakness in the calf muscles is to have patients perform a unilateral heel rise while standing flat-footed on one foot only. They may balance themselves with a hand, but most of their weight should be on that one foot. Then have the patient switch feet and have them perform the same test with the opposite foot. In general, most emergency physicians cannot use their arms to demonstrate calf weakness. However, this maneuver uses the patient’s body weight and is very good at bringing out more subtle weakness. In peripheral neuropathies the reflexes that are involved with the specific nerve are generally diminished or absent. Otherwise, they will be normal. For central lesions, the patient may have hyporeflexia to areflexia although they may also be hyperreflexic and demonstrate clonus. Other important areas to evaluate are continence of urine and feces and impotence. Urinary and fecal incontinence are almost always seen with a central lesion, although urinary incontinence has been associated with a variant of GBS. Occasionally, arm weakness or numbness from peripheral neuropathy or radiculopathy can be difficult to distinguish from stroke. As a rule, strokes do not cause limb pain,

so associated pain typically excludes stroke as a cause. Paresthesias are common in radiculopathy and peripheral neuropathy but uncommon in stroke, which usually involves loss of sensation rather than hyperesthesias or paresthesias.

Case Presentations ■ Case One A 4-month-old girl is brought in by her parents who are concerned because the child has been feeding poorly for the past several days. Her mother says that she switched from breast to bottle feeding 2 weeks ago and that her daughter “just doesn’t seem interested in eating.” Both parents state that the child has had one bowel movement in the past week, which is unusual for her, but they attributed that to the change in feedings. Other than being slightly less active than normal, there are no other complaints. The child has an uncomplicated birth history and has been healthy since her delivery. There are no sick contacts in the household, and the child stays at home with her mother during the day. Her immunizations are current. Vital signs are heart rate 132, respiratory rate 22, rectal temperature 37.3°C (99.2°F), and pulse oximetry 96% on room air. Physical examination reveals an ill-appearing child, lying quietly in her mother’s arms. The child is drooling and has some pooling of secretions in her mouth. She has coarse lung sounds,

but she is not tachypneic or retracting. Her abdomen is soft and nontender. Most notably, the child has poor muscle tone and does not resist any part of the examination. She does not interact at all with you or her parents. ■ Case Two A 24-year-old woman presents complaining of difficulty walking up stairs for the previous 2 days. Her symptoms started gradually but became more noticeable while walking up the stairs in her twostory home, especially on the day of presentation. She denies headache, back pain, fever, chills, trauma, paresthesias, and previous similar symptoms. Her medical, surgical, and family histories are all unremarkable. She takes no medications and denies any recreational drug use. On examination, she is afebrile with normal vital signs. Her physical examination is normal with the exception of her neurologic examination. Cranial nerve findings are normal, as are strength, sensation, and 2+ reflexes of the upper extremities. However, she has 5-/5 strength of her lower extremities and is unable to perform a heel rise test with either foot while the contralateral leg is lifted off the ground. She is also unable to rise from a squatting position without assistance. Her Achilles and patellar reflexes are absent and without clonus. Her sensory examination is slightly diminished diffusely in a nondermatomal distribution.

20


Critical Decisions in Emergency Medicine / Neurologic Edition

She states that she is able to feel everything on her legs but that the sensation on her legs is “just not normal.” Her Babinski examination is normal with toe flexion. Results of her laboratory evaluation (urinalysis, pregnancy test, CBC, electrolytes, calcium, magnesium, phosphate, and renal function) are normal. ■ Case Three A 35-year-old man presents at 3 am because of pain and tingling in his right hand and wrist that have been keeping him awake at night for the past several weeks. His symptoms are intermittent but are worse when he is driving or working at his computer and at night. He also tells you that the grip in his right hand is noticeably weaker than in his left. He denies neck pain or other neurologic complaints. He denies any history of traumatic injury to his wrist, hand, or spine. His past medical history is significant for insulin-dependent diabetes mellitus. The only medication he uses is insulin, and he denies any over-the-counter medication or supplement use. His vital signs are blood pressure 155/85, pulse rate 72, respiratory rate 12, temperature 37°C (98.6°F), and pulse oximetry 99% on room air. Physical examination reveals a mildly overweight man in no acute distress. His cardiovascular, respiratory, and abdominal examinations are unremarkable. There is no tenderness or pain with range-of-motion to the neck, right shoulder, or right elbow. Tinel sign (production of paresthesias in the thumb, index finger, and middle finger while tapping on the volar aspect of the wrist) and Phalen sign (production of paresthesias in the thumb, index finger, and middle finger when placing the dorsal aspect of both hands together while wrists are flexed) are both positive on the right. Opposition, abduction, and flexion of the thumb are weakened.

Botulism Botulism is a paralytic disease caused by the bacteria Clostridium

botulinum, which produces a heatstable neurotoxin. There are three distinct varieties of the disease, food-borne, wound, and infantile, the last of which has more recently been termed intestinal botulism. All have similar clinical manifestations, but the method of acquisition and the populations affected by each are unique. This disease is relatively uncommon, with only 171 cases reported in 2006 to the Centers for Disease Control and Prevention (CDC).1 Because of its rarity, the disease is frequently confused with other neurologic and infectious diseases. Despite the fact that it is a rare disease, it has such significant morbidity that most patients present initially to an emergency department for evaluation and management of their symptoms. Pathophysiology Botulism’s effect on nerve terminals is the same in all three varieties. After the toxin has entered the bloodstream, it is taken up at the presynaptic nerve terminals by endocytosis. Through a chain reaction of biochemical events, the toxin inhibits the release of acetylcholine from the presynaptic terminal. This action essentially limits motor and autonomic neuron function.2,3 Because the effects of the toxin are irreversible, clinical recovery does not occur until new neuromuscular junctions are created. Infantile (intestinal) botulism is the most common form of the disease and accounts for approximately two-thirds of all cases.1 It is caused by the ingestion of botulism spores, which then germinate, colonize, and produce toxin in an infant’s colon.4 The disease is classically associated with feeding honey to infants, but only 15% of recent cases have been linked to honey.1 The source of the remaining cases has not been identified, but the ingestion of environmental agents such as contaminated soil and vacuum cleaner dust is thought to be a likely

cause.3,5 This disease primarily affects infants, because their gastrointestinal tracts have yet to be colonized by the competitive flora that protect adults and older children.5 Food-borne botulism is caused by the ingestion of preformed toxin and is responsible for approximately 20% of cases.1 Reports are clustered in the western states, particularly Alaska, where improper canning techniques and failure to cook homecanned food properly are blamed. Occasionally, commercially produced food products are responsible. In the summer of 2007, four cases of food-borne botulism were reported. All were connected to the ingestion of Castleberry’s Hot Dog Chili Sauce, and each of those cases presented to a different emergency department.6 Wound botulism is the rarest of the three entities.1 It classically occurs when a wound becomes contaminated with Clostridia-infected soil. The bacteria then flourish and produce the toxin, which is systemically absorbed.2 Clostridia-contaminated wounds usually do not appear as grossly infected as one might expect. In the United States, cases of wound botulism are almost exclusively found in injection drug users. Recently, a cluster of cases was associated with “black tar” heroin use.7 CRITICAL DECISION What clinical clues suggest the diagnosis of botulism?

The incubation period of infantile (intestinal) botulism is nearly impossible to pinpoint. Constipation is often the initial symptom and can be present for days before neurologic symptoms occur.2 Cranial nerves are affected first, resulting in the loss of facial expression and decreased suck and cry. Initially, these findings are very subtle. As neurologic deterioration continues, the baby will develop poor head control and diffuse hypotonia (“floppy baby”).4 Symptoms progress over hours to days (mean of 4.2 days) before the child is hospitalized and the disease is

21


Critical Decisions in Emergency Medicine / Neurologic Edition

recognized.5 As in adults, respiratory paralysis is responsible for botulism mortality. Infantile botulism is thought to be an occasional etiology for sudden infant death syndrome.5 Food-borne and wound botulism have indistinguishable symptomatology. The incubation period for food-borne botulism is between 12 and 72 hours.1 Distinguishing a precise incubation period for wound botulism is problematic, if not impossible. Symptoms of both conditions usually start with cranial nerve deficits; diplopia, facial weakness, dysphagia, ptosis, and speech changes are all common findings. Paralysis then descends, affecting the upper extremities before the lower. Respiratory muscle paralysis occurs as the process descends. The autonomic nervous system is also affected, resulting in dry mouth, postural hypotension, paralytic ileus, and pupillary abnormalities. In one emergency department case series on food-borne botulism, all 29 patients had at least three of the following: weakness, dry mouth, double vision, and difficulty speaking.8 In a larger case series of 705 patients, 68% had at least three of the following symptoms on admission: nausea and vomiting, dysphagia, diplopia, dry mouth, and fixed and dilated pupils.2 CRITICAL DECISION Are there any confirmatory tests for botulism that can be performed in the emergency department?

In all three varieties of botulism, the diagnosis is clinical. Stool, wound, and blood for toxin and culture should be tested; however, these are of no help in the emergency department diagnosis and management of the condition. Routine diagnostic studies are only helpful to rule out other conditions. The best way to confirm the diagnosis of infantile botulism is through identification of the toxin in the stool, which may be difficult as these patients are usually very constipated.5

CRITICAL DECISION What immediate life threats must be addressed in a patient with botulism?

All patients with suspected botulism should be admitted to the ICU for observation and supportive care. The primary concern is the evaluation and management of the patient’s airway and respiratory status. If there is any concern for airway or respiratory compromise, the patient should be immediately intubated and maintained on mechanical ventilation until the toxin’s effects have worn off, which could take weeks. Even if the patient’s respiratory status is good on initial evaluation, it must be re-assessed frequently, because this disease can progress rapidly. CRITICAL DECISION Other than supportive care, what can be done for a patient with botulism?

In addition to supportive care, all patients with food-borne and wound botulism should be treated with one vial of botulism antitoxin.1 Because this is derived from horse serum, all patients should have skin testing for hypersensitivity before administration. The antitoxin has been shown to decrease mortality rates and reduce hospital length-ofstay. Although it prevents progression of the disease, it does not reverse the paralysis—the toxin binding is irreversible. The antitoxin can be obtained through state and local health departments and the CDC.1 In patients with wound botulism, the wound should be débrided. Antibiotics should be given for coexisting infection, but no studies have shown that their administration hastens recovery from the paralysis. For infantile botulism, supportive care is, again, the backbone of treatment. Antibiotics are ineffective.5 The antitoxin is not used in infants out of concern for reactions to the horse serum derivative. Recently, human botulism immune globulin (HBIG) has been made available by the FDA for infantile botulism

only. Like the antitoxin, it has been shown to decrease ICU length of stay, ventilation requirements, and mortality rates.9 It can be obtained by calling the Infant Botulism Treatment and Prevention Program at 510-231-7600.

Guillain-Barré Syndrome GBS is the most common acute noncompressive motor neuropathy.10-12 It typically occurs in otherwise healthy people and in men about 1.5 times more commonly than in women.13 The incidence of GBS increases linearly with age.13 In the classic presentation, GBS is preceded by a prodromal event that occurs approximately 1 to 4 weeks before the onset on symptoms. The most commonly identified precipitant is gastroenteritis due to Campylobacter jejuni.10,13-15 However, it is also associated with nonspecific viral upper respiratory infections, surgery, vaccination, and herpes, mycoplasma, and HIV infection.10,13,14,16,17 Often, no clear etiology is present, so the absence of a prodromal event is not sufficient evidence to rule out this syndrome. The more severe or advanced forms of GBS can be easily confused with an epidural compression syndrome or transverse myelitis because of the symmetrical features and the significant motor weakness. Sensory deficits associated with GBS; however, are usually much less pronounced than those associated with epidural compression syndrome or transverse myelitis. Additionally, clonus will be absent and the Babinski examination will be negative for GBS. CRITICAL DECISION A patient who has motor weakness also has sensory deficits and complains of pain; does this rule out Guillain-Barré syndrome (GBS)?

GBS is an acute inflammatory demyelinating polyradiculoneuropathy.10-13 It is not a specific disease, but rather it is a heterogenous group of immunemediated peripheral neuropathies that involve the motor, sensory, 22


Critical Decisions in Emergency Medicine / Neurologic Edition

and autonomic nerves.10,11,13 The inflammation from GBS causes a disruption or stripping of myelin from the peripheral nerve axons, which results in diminished nerve conduction.11,13,15,17,18 Interestingly, although GBS is classically described as affecting only the peripheral nerves, it seems to have a predilection for the spinal nerve roots, and in rare cases it can involve the central nervous system to a minor degree.11 In most cases, the net result is a loss of muscular innervation as well as some sensory deficits. The sensory deficits generally do not follow a dermatomal distribution but are usually mild and may include paresthesias of the hands and feet, especially at the beginning of the weakness.11 Pain, which can be very severe in some cases, occurs in up to 89% of cases of GBS. The type of pain varies, but various forms include dysesthesias, a radicular pain in a bilateral sciatic distribution, and an aching in the large muscles of the legs, thighs, or back similar to a “charley horse.”11,13 In addition to the classic GBS, there are several variants that emergency physicians must be aware of, as follows10-13,15-17,19,20: • Miller-Fisher variant: Ophthalmoplegia, areflexia, and ataxia. • Descending motor weakness: Motor weakness that involves the cranial nerves first and then descends to involve the arms, then the legs. This occurs in up to 14% of cases. • Pure sensory GBS: An ascending sensory loss with diminished or absent reflexes but without any motor loss. • Pure pandysautonomia: Autonomic dysfunction and areflexia without motor or sensory loss. This is uncommon and unlikely to be diagnosed in the emergency department.

CRITICAL DECISION Should all patients with suspected GBS undergo a lumbar puncture in the emergency department?

GBS requires a high degree of suspicion if one is to diagnose patients presenting early in the disease process, which can be life saving. Most patients are afebrile, and at first glance they appear well and have a normal status. It is the progressive nature of the weakness and other neurologic symptoms over a period of days that should intimate this diagnosis, unless the patient has a more severe and aggressive form of the disease. With bilateral symmetric motor weakness and hyporeflexia to areflexia, an emergency physician should suspect GBS and seek to confirm or exclude the disease. Because results of typical blood tests such as the CBC and serum electrolytes will be normal, there is little left in an emergency physician’s armamentarium to assist in making the diagnosis. In this case, lumbar puncture and examination of the spinal fluid might be helpful. The classic cerebrospinal fluid (CSF) finding in patients with GBS is an elevated protein with none or only a few CSF WBCs, a feature called albuminocytologic dissociation. However, CSF protein concentrations are commonly normal in the first week, increasing by the end of the second week of the disease process,11,13 so a normal CSF examination cannot be used to rule out this diagnosis.11,13,21 Emergency physicians may perform a lumbar puncture to assist them in making the diagnosis, especially if symptoms are consistent with GBS. CRITICAL DECISION Do all patients with suspected GBS require hospital admission, and if so should it be to the ICU?

If an emergency physician suspects GBS, the patient requires hospital admission. This allows for a more definitive evaluation, monitoring of the patient’s respiratory and neurologic status, and further

treatment. The most prudent approach is to have a neurologist evaluate the patient in the emergency department, because these patients can rapidly decompensate. Recognizing that a neurologist is not available in all emergency departments, the next best approach would be to transfer the patient to a hospital that has a neurologist and an ICU. Because these patients are at such high risk of rapid disease progression and decompensation, emergency physicians should not admit these patients to the ward with holding orders. All of these patients should be evaluated by their admitting physician in the emergency department. In the hospital, the patient is usually evaluated further using electrodiagnostic studies such as an electromyelogram or nerve conduction velocity testing. In the emergency department, measure a forced vital capacity (FVC) to help determine the degree of respiratory muscle involvement, which will assist in admitting the patient to the appropriate setting in the hospital. A normal FVC is approximately 65 mL/kg.11 If the FVC is less than one-half of the predicted FVC, then the patient should be admitted to the ICU and could require endotracheal intubation and mechanical ventilation.22 Other predictors of the potential need for endotracheal intubation and mechanical ventilation include the following: 1) time from GBS onset to hospital admission of less than 7 days, 2) inability to lift the elbows or head above the bed, 3) inability to stand, 4) ineffective coughing, and 5) increased liver enzymes.10 In the hospital, the patient will be treated with either intravenous immunoglobulin (IVIG) or plasmapheresis but not both together. There is no proven benefit to treatment with steroids.10-13 The clinical course of the disease progresses over 2 to 4 weeks, with most patients reaching the nadir of their weakness by 2 to 3 weeks.10,11,13,17 Of all patients with GBS, 25% to 30% will require 23


Critical Decisions in Emergency Medicine / Neurologic Edition

ventilatory support, and 3% to 8% will die.10,11,13,17,22 Common etiologies for death are cardiac arrest (most likely due to dysautonomia), sepsis, acute respiratory distress syndrome, and pulmonary embolism.10-13 Unfortunately, 3% of patients will have a recurrence of their disease.10,13

Carpel Tunnel Syndrome Carpal tunnel syndrome is the most common entrapment neuropathy, with a lifetime incidence of approximately 3% in the United States.23 Most cases of carpal tunnel syndrome are idiopathic, but pregnancy, hypothyroidism, diabetes, renal failure, acromegaly, and steroid use are all thought to increase risk.24 Occupations that involve repetitive flexion at the wrist such as typing, logging, and construction work can also lead to carpal tunnel syndrome. Additionally, women are three times more likely to get carpal tunnel syndrome than men. Pathophysiology Carpal tunnel syndrome results from compression of the median nerve as it passes through the carpal tunnel, which is formed by the rigid carpal bones, nine flexor tendons, and the transverse carpal ligament.23,25 This causes pain, paresthesias, and weakness in the median nerve distribution. Diagnosis Carpal tunnel syndrome most commonly presents with pain, tingling, burning, or numbness in the median nerve distribution. This includes the volar aspects of the thumb, index, and middle finger and the radial half of the ring finger. Symptoms usually worsen with activity and commonly wake the patient from sleep at night. Patients frequently report shaking their affected hand when the pain is present in an attempt to rid themselves of the discomfort. Carpal tunnel syndrome can be bilateral, but the symptoms are usually more severe in the dominant hand. On examination, sensory deficits

to light touch and two-point discrimination can be detected. Weakness of thumb adduction and thenar atrophy are found in more advanced cases. Phalen maneuver and Tinel sign (both described previously) both suggest carpal tunnel syndrome when positive. The sensitivity of each test is poor (25% to 50% for Tinel and 10% to 91% for Phalen).23,24 Electrodiagnostic testing can be performed on an outpatient basis to confirm the diagnosis. CRITICAL DECISION What is the most effective, nonsurgical treatment modality for carpal tunnel syndrome?

Placement of a neutral wrist splint is effective in mild cases and is the least invasive treatment option.26 Splinting is most effective when used continuously, although many patients choose to use splints only at night. Even though nonsteroidal antiinflammatory drugs are commonly prescribed, studies show that they are ineffective in the management of carpal tunnel syndrome.24 Steroid injections into the carpal tunnel can relieve symptoms in the short term,27 but surgery is the definitive treatment in refractory cases.28

Case Resolutions â– Case One The infant who was not feeding normally was recognized to have an at-risk airway and was immediately intubated. A sepsis workup was initiated, including a CBC, urinalysis, blood and urine cultures, a chest radiograph, and a lumbar puncture. Results of a thorough ophthalmoscopic examination and a noncontrast computed tomography scan of the head were normal. The child was empirically started on vancomycin, ceftriaxone, and acyclovir and admitted to the pediatric ICU for further management. One day after admission, all cultures were negative, and infantile botulism was considered. Minimal stool was obtained and sent to the

state laboratory, where it was found to be positive for botulinus toxin. Her providers called the Infant Botulism Treatment and Prevention Program and obtained HBIG, which was administered at a dose of 50 mg/kg. She had a prolonged period of mechanical ventilation but was eventually discharged home with no residual morbidity. â– Case Two A lumbar puncture was performed in the emergency department on the young woman who was having difficulty walking up stairs, and the results were completely normal. She was admitted to the hospital to a monitored care setting by the neurology service. On the first day of her admission, she had nerve conduction studies, results of which were consistent with GBS. She was treated with IVIG and had a slight progression of her weakness but only to the level of her legs to the extent that she needed assistance to walk. Three weeks after admission, she had improving strength and was discharged home. No etiology for her GBS was ever determined. â– Case Three The man with severe hand and wrist pain had a thorough examination in the emergency department, which led to the clinical diagnosis of carpal tunnel syndrome. Radiographs of his wrist and hand were read as normal. He was placed in a neutral wrist splint that he wore continuously for several weeks. When his symptoms did not completely resolve he was operated on by a hand surgeon.

Summary Emergency physicians have the difficult job of rapidly evaluating and managing disease that may be presenting early in the disease process. Through diligence and attention to detail, emergency physicians should be able to identify the life-threatening neuropathies of botulism and GBS. Once either of these diseases is suspected, the 24


Critical Decisions in Emergency Medicine / Neurologic Edition

Pearls • Consider the diagnosis of botulism in toxicappearing infants. • All patients with suspected botulism should be admitted to the ICU. • Findings that suggest the diagnosis of botulism are dry mouth, double vision, difficulty speaking, dysphagia, diplopia, and fixed and dilated pupils. • Contact the CDC or local health department to obtain HBIG or botulism antitoxin as soon as the diagnosis is suspected. • Be diligent in performing the neurologic examination in patients who present with bilateral lower extremity symptoms. • A lumbar puncture can be normal in a patient who is in the early stages of GuillainBarré syndrome (GBS). • GBS patients with an FVC less than 50% of predicted should be admitted to the ICU. • Placement of a neutral wrist splint is the most effective initial treatment for carpal tunnel syndrome.

Pitfalls • Not identifying subtle weakness by conducting a squat down or heel rise test. • Failing to initiate rapid airway control and respiratory support in a patient who is deteriorating secondary to presumed botulism. • Admitting a patient with suspected GBS to the ward with “holding” orders and without close observation. • Relying on normal laboratory test results, including a lumbar puncture, to rule out GBS. • Treating carpal tunnel syndrome with a dorsiflexion wrist splint, which can worsen the symptoms.

initial management decision is directed towards ensuring that the airway is protected and that the patient does not require immediate or urgent respiratory assistance. Once this life-saving intervention has been considered or implemented, emergency physicians should further evaluate these patients and involve the appropriate consultants to admit the patient to the hospital. It is important to remember that laboratory testing from the emergency department is unlikely to confirm these disease processes, although it could help rule out other etiologies for the patient’s symptoms. Carpal tunnel is the most common entrapment peripheral neuropathy, and emergency physicians will treat patients with this disorder. In managing patients with carpal tunnel syndrome, one should inform the patient that time and splinting are the best treatments short of carpal tunnel release. Finally, although nonsteroidal anti-inflammatory drugs are commonly prescribed, they are not proven to make a difference for patients. Ensure that these patients get appropriate followup for reevaluation and management, because medical treatment is limited.

12. Hartung HP, Kieseier BC, Kiefer R. Progress in GuillainBarré syndrome. Curr Opin Neurol. 2001;14(5):597-604. 13. Van Doorn PA, Ruts L, Jacobs BC. Clinical features, pathogenesis, and treatment of Guillain-Barré syndrome. Lancet Neurol. 2008;7:939-950. 14. Tsang RS. The relationship of Campylobacter jejuni infection and the development of Guillain-Barré syndrome. Curr Opin Infect Dis. 2002;15(3):221-228. 15. Ho T, Griffin J. Guillain-Barré syndrome. Curr Opin Neurol. 1999;12(4):389-394. 16. Pascuzzi RM. Peripheral neuropathies in clinical practice. Med Clin North Am. 2003;87(3):697-724. 17. Ayyar R. Clinical presentations of peripheral neuropathies. Neuroimaging Clin North Am. 2004;14(1):55-58,vii. 18. Hartung HP, Willison HJ, Kieseier BC. Acute immunoinflammatory neuropathy: update on GuillainBarré syndrome. Curr Opin Neurol. 2002;15(5):571-577. 19. Freeman R. Autonomic peripheral neuropathy. Neurol Clin. 2007;25(1):277-301. 20. Oh SJ, LaGanke C, Claussen GC. Sensory GuillainBarré syndrome. Neurology. 2001;56(1):82-86. 21. McGillicuddy DC, Walker O, Shapiro NI, Edlow JA. Guillain-Barré syndrome in the emergency department. Ann Emerg Med. 2006;47(4):390-393. 22. Sharshar T, Chevret S, Bourdain F, et al. Early predictors of mechanical ventilation in Guillain-Barré syndrome. Crit Care Med. 2003;31(1):278-283. 23. D’Arcy CA, McGee S. The rational clinical examination. Does this patient have carpal tunnel syndrome? JAMA. 2000;283:3110-3117. 24. Katz JN, Simmons BP. Clinical practice. Carpal tunnel syndrome. N Engl J Med. 2002;346:1807-1812. 25. Hochman MG, Zilberfarb JL. Nerves in a pinch: imaging of nerve compression syndromes. Radiol Clin North Am. 2004;42:221-245. 26. Goodyear-Smith F, Arroll B. What can family physicians offer patients with carpal tunnel syndrome other than surgery? A systematic review of nonsurgical management. Ann Fam Med. 2004;2:267-273. 27. Dammers JW, Veering MM, Vermeulen M. Injection with methylprednisolone proximal to the carpal tunnel: randomised double blind trial. BMJ. 1999;319:884-886. 28. Hui AC, Wong S, Leung CH, et al. A randomized controlled trial of surgery vs steroid injection for carpal tunnel syndrome. Neurology. 2005;64:2074-2078.

References 1. Centers for Disease Control and Prevention. Emergency Preparedness and Response. Botulism: Information and Guidance for Clinicians Online. Available at: http://www.bt.cdc.gov/agen/botulism/ clinicians. Accessed August 31, 2009. 2. Greenberg D, Aminoff M, Simon R, eds. Clinical Neurology. 5th ed. Chicago, IL: McGraw-Hill; 2002. 3. Lawrence DT, Dobmeier SG, Bechtel LK, Holstege CP. Food poisoning. Emerg Med Clin North Am. 2007;25(2):357-373; abstract ix. 4. Schreiner MS, Field E, Ruddy R. Infant botulism: a review of 12 years’ experience at the Children’s Hospital of Philadelphia. Pediatrics. 1991;87(2):159-165. 5. Long SS. Infant botulism. Pediatr Infect Dis J. 2001;20(7):707-709. 6. Botulism associated with commercially canned chili sauce—Texas and Indiana, July 2007. MMWR Morb Mortal Wkly Rep. 2007;56(30):767-769. 7. Passaro DJ, Werner SB, McGee J, et al. Wound botulism associated with black tar heroin among injecting drug users. JAMA. 1998;279:859-863. 8. Ruthman JC, Hendricksen DK, Bonefeld R. Emergency department presentation of type A botulism. Am J Emerg Med. 1985;3:203-205. 9. Arnon SS, Schechter R, Maslanka SE, et al. Human botulism immune globulin for the treatment of infant botulism. N Engl J Med. 2006;354:462-471. 10. Newswanger DL, Warren CR. Guillain-Barré syndrome. Am Fam Physician. 2004;69(10):2405-2410. 11. Ropper AH. The Guillain-Barré syndrome. N Engl J Med. 1992;326(17):1130-1136.

25


Critical Decisions in Emergency Medicine / Neurologic Edition

21. Which one of the following tests best confirms the diagnosis of infantile botulism? A. Blood culture B. Cerebrospinal fluid analysis C. Lung biopsy for botulism culture D. Stool sample demonstrating botulinum toxin

28. Which one of the following is a laboratory finding consistent with Guillain-BarrĂŠ syndrome (GBS)? A. Elevated protein with no WBCs in the cerebrospinal fluid (CSF) B. Elevated protein with over 1,000 RBCs in the CSF C. Elevated WBC count in the blood D. Elevated WBC count in the CSF

22. What treatment for botulism has been shown to decrease hospital length of stay? A. Broad-spectrum antibiotics B. Gastric lavage C. Human botulism immunoglobulin (HBIG) D. Mechanical ventilation

29. Which of the following are findings seen with classic GBS? A. Fever and severe back pain B. Paralysis and numbness of one leg in a solitary nerve root distribution C. Paralysis starting with the face and moving down the body D. Paresthesias of the hands and feet, especially at the onset of leg weakness

23. What is the first neurologic deficit most commonly seen in patients with botulism? A. Ataxia B. Cranial nerve deficits C. Lower extremity weakness and numbness D. Respiratory paralysis 24. What is the most common variety of botulism in the world? A. Disseminated botulism B. Food-borne botulism C. Infantile botulism D. Pulmonary botulism

30. Which one of the following predicts the likely need for endotracheal intubation in a patient with GBS? A. An elevated CSF protein B. A forced vital capacity of greater than 65 mL/kg C. An inability to lift the head or elbows above the bed without assistance D. A low WBC count from the blood

25. What is the first, critical step that must be taken in managing a patient with suspected botulism? A. Airway management and respiratory support if needed B. Fluid resuscitation C. Immediate isolation precautions D. Parenteral benzodiazepines to prevent seizures 26. Which of the following symptoms are seen in patients with carpal tunnel syndrome? A. Decreased sensation in the small finger and the ulnar half of the ring finger B. Decreased sensation in the small finger and the ulnar half of the ring finger along with hypothenar atrophy C. Decreased sensation in the thumb, index, and middle fingers and the radial half of the ring finger D. Hypothenar atrophy and weakness of wrist flexion 27. What is the definitive therapy for refractory carpal tunnel syndrome? A. Carpal tunnel release B. Neutral wrist splint C. Nonsteroidal anti-inflammatory drugs D. Opiate analgesics

26


Critical Decisions in Emergency Medicine / Neurologic Edition

Evaluation and Management of Stroke Lesson 4

Brian E. Burgess, MD, FACEP, and Justin Stowens, MD

■ Objectives On completion of this lesson, you should be able to: 1. Classify the various stroke syndromes and describe their presentations. 2. Define the evaluation of the stroke patient, including use of the National Institutes of Health Stroke Scale. 3. Explain the benefits and limitations of currently available and commonly used imaging modalities. 4. Understand the risks and benefits of the various treatment regimens for patients with acute stroke. 5. Recognize patients who present with stroke mimics. 6. Describe the complications that can arise from the primary stroke process.

■ From the EM Model 12.0 Nervous System Disorders 12.11 Stroke (Cerebral Vascular Events)

■ Addresses this state-specific CME requirement: • Stroke

The incidence of stroke is significant, affecting 795,000 Americans annually. Every 40 seconds a stroke develops, resulting in death every 4 minutes.1 Stroke is the fourth most common cause of death in the United States, and 55% of all patients who survive their stroke are discharged to skilled nursing facilities or inpatient rehabilitation centers.1 Emergency physicians are uniquely positioned to treat a stroke patient early during the evolution of a stroke. The diagnosis can be difficult to determine in the face of varying clinical presentations and stroke mimics; however, accurate recognition and treatment often determine the patient’s ultimate outcome.2

Case Presentations ■ Case One A 58-year-old man arrives by EMS 2 hours after he suddenly began to experience difficulty speaking and numbness in his right hand and arm. His wife noticed that he was having difficulty using his dinner utensils. His past medical history is notable for atrial fibrillation, hypertension, and chronic obstructive pulmonary disease. He takes metoprolol and albuterol daily; he stopped taking warfarin 1 week ago in preparation for inguinal hernia surgery scheduled for 3 days from today. His initial vital signs are blood pressure 205/100, pulse rate 58, respiratory rate 20, temperature 37.2°C (99°F), and pulse oximetry 98% on 2 liters of oxygen via nasal cannula. Intravenous catheters have

been inserted, and his initial ECG shows atrial fibrillation with a rate of 50 to 60. His airway is intact; his breathing is unlabored, and he is able to follow commands despite some expressive aphasia. He denies any recent trauma or falls as well as any alcohol or illicit drug use. He confirms that he has never had a stroke in the past. He is alert but has trouble naming the current month or his name. Cranial nerves II-XII are intact grossly, and there are no visual field deficits or gaze palsy. He has weakness of the right lower extremity but is able to lift it briefly off of the stretcher. He cannot move his right hand or arm and has mild sensory loss in the right upper extremity but shows no signs of hemineglect, inattention, or dysarthria despite his aphasia. He received a noncontrastenhanced computed tomography (NECT) scan and was returned to the emergency department without a change in condition; the NECT is read by the radiologist as normal. An acute occlusion of the left middle cerebral artery (MCA) is suspected based on the constellation of symptoms. The team gathers to discuss the next steps in this patient’s treatment.

■ Case Two

A 65-year-old man presents with a migraine headache that started abruptly 1 hour ago, after he ingested chocolate. He has a history of migraine and tension headaches as well as high blood pressure, myocardial infarction (MI), and coronary artery bypass 27


Critical Decisions in Emergency Medicine / Neurologic Edition

Critical Decisions • What clinical features can help emergency physicians in diagnosing a stroke?

• How should emergency physicians manage complications from thrombolytic use?

• What are the important steps in the early stabilization and management of patients having a stroke?

• What is the role of endovascular intervention in the treatment of stroke?

• Which imaging modality is most useful in the evaluation of a stroke patient?

• What is the appropriate emergency department management of stroke caused by spontaneous intracerebral hemorrhage?

• How should thrombolytic therapy be implemented in patients with acute ischemic stroke?

grafting 15 years ago. He has a mechanical heart valve for which he takes dabigatran daily. He also takes metoprolol, lisinopril, furosemide, and oxycodone. He reports that the occipital location of his headache is atypical, but that it was preceded by his usual aura and visual scotoma. His vital signs are blood pressure 150/80, pulse rate 60, respiratory rate 20, temperature 37.1°C (98.7°F), and pulse oximetry 98%. He is given hydromorphone, 0.5 mg IV, ondansetron, 4 mg IV, and normal saline, 150 mL/hour, and placed in a darkened examination room. Several minutes later he is found unresponsive with a nonreactive, dilated right pupil, snoring respirations, and blood pressure of 220/140. After escalating doses of naloxone elicit no response, the patient is intubated using rapid sequence intubation (RSI). Routine blood work is ordered STAT, including coagulation studies. His blood is typed and cross-matched for 4 units of packed red blood cells. The subsequent NECT reveals a large area of intraparenchymal hemorrhage in the right occipital and parietal lobes with mass effect and resulting midline shift of 15 mm. The patient’s family is notified, and a request for neurosurgical consult is placed. Strokes have traditionally been classified two ways—either based on the mechanism of the stroke or on the anatomy affected. Both

• What is the appropriate disposition for stroke patients after treatment in the emergency department?

classifications help direct diagnostic and treatment strategies. Within the mechanism classification, strokes can be divided broadly into hemorrhagic and ischemic. Excluding traumatic hemorrhage, spontaneous hemorrhagic stroke is further divided into intracerebral hemorrhage (ICH) and subarachnoid hemorrhage (SAH). ICH tends to provide a focal yet gradual presentation over minutes to hours and usually involves a progression of symptoms. Severe headaches associated with vomiting, more common with ICH or SAH, are not generally a feature of the ischemic strokes. Both classes of hemorrhagic strokes are often associated with strenuous activity. ICH that is associated with oral anticoagulants is often large and rapidly expanding. ICH patients presenting with diminished alertness on anticoagulation have higher mortality rates than those who are not on anticoagulants. Subtypes of ischemic stroke are embolic, thrombotic, and hypoperfusion-induced stroke. Most thrombotic strokes occur in older patients with atherosclerosis; however, patients in their 40s and 50s can present with a thrombotic stroke, particularly if they have risk factors such as hypertension, diabetes, smoking, hyperlipidemia, and a strong family history. Embolic strokes are often seen in patients younger than 50 years and in those with new-onset atrial fibrillation, valvular

heart disease, or endocarditis. They generally occur suddenly and have maximal deficits at the onset. Multiple sites can be affected, particularly if the source is aortic or cardiac. Sudden ischemia with a pale cold extremity suggests an embolic cause. Atrial fibrillation, heart sounds consistent with mechanical heart valve replacement, recent MI, and heart murmurs associated with aortic and mitral valve disease suggest a cardiac source for an embolic stroke. Fever or signs of infective endocarditis also suggest a cardiac origin for an embolic stroke. Strokes caused by hypoperfusion are especially challenging as they present with multiple problems for the emergency physician to address. Namely, the physician must address both the neurologic insult as well as the underlying problem that is causing the hypoperfusion. The hypoperfusion injury can initially present as a focal stroke rather than with a bilateral or diffuse injury pattern because of disproportionate unilateral atherosclerosis. Finally, chronic hypertension is frequently associated with smaller ischemic strokes called lacunar infarcts. These small areas of infarction often result in pure motor or pure sensory deficits. Thrombotic strokes can involve large arteries (carotids, vertebral arteries, the Circle of Willis and its branches). These strokes tend to be recurrent and fluctuating and can

28


Critical Decisions in Emergency Medicine / Neurologic Edition

evolve over several days. Smaller vessel thrombotic strokes involve the penetrating arteries that affect the deeper areas of the brain such as the internal capsule, basal ganglia, thalamus, and pons. Absent or diminished pulses in the extremities or proximal vasculature and carotid bruits suggest atherosclerotic disease and a thrombotic cause. Venous occlusion, a rarer cause of stroke, can impair the drainage of blood from the brain resulting in ischemia or hemorrhage. Although both ischemia and hemorrhage ultimately lead to the end point of poorly perfused brain and the observed neurologic deficits, there are differences in the patient’s history and presentation that help differentiate these at the bedside.

CRITICAL DECISION What clinical features can help emergency physicians in diagnosing a stroke?

Various regional stroke syndromes have been described by their unique clinical features and can be divided into four main anatomic locations, the cerebral cortex, the pons and midbrain, the brainstem, and the cerebellum. Stroke affecting the cerebral cortex results in motor and sensory deficits involving the contralateral face, arm, and leg. Pontine strokes often present with miosis, abnormal breathing patterns, gaze paralysis, and coma. Brainstem strokes involve ipsilateral facial weakness with contralateral arm and leg weakness. Cerebellar strokes can present with vertigo,

ataxia (gait and truncal), drop attacks, nystagmus, severe nausea, and vomiting. Stroke location can be divided into anterior (carotid) and posterior (vertebrobasilar) circulation strokes. Classification schemes vary, but almost all include a variation resembling Bamford’s analysis of data from the Oxfordshire Community Stroke Project.3 The lack of anatomic correlation raises the possibility of a stroke mimic. Stroke mimics include tumor, seizure, migraine aura, infection, metabolic disturbances, syncope, head injury, multiple sclerosis, vasospasm, and functional disorders. Some specific vessel occlusions have classic presentations that are useful to recognize. Involvement of the anterior cerebral artery can

Table 1. National Institutes of Health Stroke Scale 1a. Level of consciousness (LOC)

1b. LOC questions Ask patients their age. Ask the patient to name the current month. Coma score = 2a 1c. LOC commands Ask patients to open and close their eyes. Ask patients to squeeze and release their non-paralyzed hand. Coma score = 2 2. Best gaze Patient follows finger horizontally. Coma score = must be tested by rotating the patient’s head and assessing for oculocephalic reflex (a present reflex is indicated by maintenance of a forward gaze). 3. Visual Tested by confrontation with stimulus to all 4 quadrants of both eyes. Coma score = must be tested by visual threat. 4. Facial palsy Ask/pantomime patient to smile, raise eyebrows. Coma Score = test with noxious stimulus. 5a and b. Motor arm (for each arm) Have patient extend arm, with palm down, 45° and hold for 5 seconds. Coma Score = 8

Alert Drowsy Stuporous Coma Answers both correctly Answers one correctly Answers both wrong Performs both correctly Performs one correctly Performs both wrong

0 1 2 3 0 1 2 0 1 2

Normal Partial gaze palsy (isolated cranial nerve III, IV, VI, or deviation is overcome by reflex or voluntary) Forced deviation

0 1

No visual field loss Partial hemianopia Complete hemianopia Bilateral hemianopia or blindness Normal Minor paralysis Partial paralysis Complete paralysis No drift Drift Some effort against gravity No effort against gravity No movement

0 1 2 3 0 1 2 3 0 1 2 3 4

2

29


Critical Decisions in Emergency Medicine / Neurologic Edition

result in sensory loss and paralysis in the contralateral lower face, arm, and leg, with greater involvement in the leg. Cognitive and emotional manifestations may be prominent. An MCA stroke can yield paralysis and sensory loss in the contralateral face, arm, and leg, with paralysis greater in the face and arm, aphasia (left MCA), or hemineglect (right MCA). With an MCA stroke, conjugate eye deviation towards the side of the brain lesion and contralateral homonymous hemianopsia may be present. Dysphagia, dyscalculia, and visual-spacial dysfunction can occur. Lacunar infarcts, affecting the small penetrating vessels of the MCA, frequently present with pure motor or pure sensory symptoms or ataxic hemiparesis. A posterior cerebral

artery stroke often yields visual field deficits, confusion, dizziness, and memory and language disorders. Cortical blindness, contralateral homonymous hemianopsias, and visual agnosia can also be present. Patients with vertebral basilar involvement may have ipsilateral cranial nerve palsy with contralateral motor and sensory deficits. Associated syncope, coma, quadriplegia, diplopia, visual field cuts, dysarthria, dysphagia, vertigo, nystagmus, and vomiting can occur. In the worst case, only the eyes are spared, the “locked in syndrome.” An objective measure of stroke severity is usually performed simultaneously with the clinical assessment. It will help in the later decision to administer or

withhold thrombolytics as well as in communicating initial findings to subsequent physicians. The neurologic examination in stroke is simplified into an easily performed and reliably reproduced series of questions, tasks, and physical findings. The most widely used examination tool is the National Institutes of Health Stroke Scale (NIHSS) (Table 1). It should be used by any physician caring for stroke patients. As of the time of this lesson, free online training in its use, with video examples, is available at http:// nihstrokescale.org. The complete examination with examples is available for free printing at http:// www.ninds.nih.gov/doctors/NIH_ Stroke_Scale.pdf. The examination score has been shown to correlate

Table 1, National Institutes of Health Stroke Scale, continued 6a and b. Motor leg (for each leg) Have the patient hold the leg flexed at the hip to 30° and hold for 5 seconds. Coma Score = 8 7. Limb ataxia Finger to nose and heel to shin testing in all four extremities Coma = 0 (as ataxia is not demonstrated in comatose patients.) 8. Sensory. Test response to pinprick or noxious stimuli in many body areas to determine presence of deficit. Coma score = 2 9. Best language. Language may be graded from responses to prior questions and interaction thus far. Or the patient can be asked to describe pictures and name objects. Coma score = 3 10. Dysarthria. Ask the patient to repeat words or phrases. With aphasia, grade the clarity of articulation of the broken phrases. Intubated = Unscorableb Coma score = 2 11. Extinction and inattention Observation for hemi-neglect can be made during the prior questions. Coma score = 2

a

No drift Drift Some effort against gravity No effort against gravity No movement Absent Present in one limb Present in two limbs No sensory loss Mild to moderate (some loss, but patient is aware of stimuli) Severe to total loss (patient is not aware at all of being touched or aware of the noxious stimuli) No aphasia Mild to moderate difficulty in communication and obvious loss of fluency. Severe aphasia resulting in communication through fragmented expression Mute, global aphasia Normal Mild to moderate; slurs some words but can be understood. Severe dysarthria; slurred as to be unintelligible or mute. No abnormality Inattention or extinction in one sensory modality (sight, touch, sound, spacial awareness, or personal inattention) Profound hemi-inattention or extinction in more than one modality

0 1 2 3 4 0 1 2 0 1 2 0 1 2 3 0 1 2 0 1

2

Several of the tasks performed in the NIHSS are not possible for comatose patients; the provided “coma score” should be used for these patients to achieve accurate reproducibility.

b

An unscorable examination is recorded as unscorable and does not add to the total score.

30


Critical Decisions in Emergency Medicine / Neurologic Edition

directly with the severity of stroke and has excellent reproducibility.4 CRITICAL DECISION What are the important steps in the early stabilization and management of patients having a stroke?

The overall goal of initial stroke management is to treat immediate life-threatening conditions and progress towards imaging, with the subsequent application of either fibrinolytics and/or intravascular intervention if available and warranted. Stabilization of patients suffering from a stroke begins with attention to the airway, breathing, and circulation (the ABCs), with close attention to abnormal vital signs. Early bedside glucose levels should be ascertained and hypoglycemia corrected accordingly. Hypoxia is extremely common in the immediate post stroke period, and if the clinician elects not to intubate the patient, close monitoring of the airway is paramount. Recognition of increased intracranial pressure (ICP) early in a patient’s presentation can prove lifesaving. Elevated ICP can occur from hemorrhage, large unilateral or bihemispheric ischemia, or vertebrobasilar ischemia, leading to brain edema. If elevated ICP is present or suspected, appropriate RSI techniques should be used to minimize increases in ICP. The body’s autoregulation of ICP is usually successful when a mean arterial pressure (MAP) of 60 to 160 mm Hg is maintained. The progression of the stroke, iatrogenic airway manipulation, and drugs administered by the physician can all elevate (or lower) the MAP outside this range, allowing reduced perfusion and continued cerebral injury. The injured brain is especially sensitive to low perfusion pressures, and maintenance of an appropriate blood pressure is vital. Airway management in such situations is essential. Briefly, when controlling the airway in a patient with suspected

increased ICP, particularly those patients who are vomiting from an intracranial bleed or vertebralbasilar ischemia, the steps of RSI remain the same. Pretreatment with lidocaine and fentanyl have been shown to blunt the reflex sympathetic response to laryngoscopy without a definitive outcome benefit and are not recommended.5 Induction agents should be chosen with respect to the patient’s overall hemodynamics. Etomidate (0.3 mg/kg IV push) can safely be used in the setting of increased ICP. Ketamine (1 to 2 mg/ kg IV) or etomidate can be used in patients who are systemically normotensive to hypotensive. Ketamine can elevate heart rate and blood pressure and should be avoided in patients with significant systemic hypertension. Controversy remains regarding the paralytic of choice because the fasciculation phase caused by succinylcholine (1.5 to 2 mg/kg IV) is associated with an increased ICP in an animal model.6 Rocuronium (1 mg/kg IV), a nondepolarizing neuromuscular blocker, does not carry the same risk; however, respiratory paralysis induced by rocuronium lasts significantly longer than that from succinylcholine—40 to 45 minutes. The decision remains the purview of the attending physician. Blood pressure management in stroke patients warrants discussion both because of the complexity of the management and because elevated blood pressure is a contraindication for fibrinolytic therapy. It is well documented that blood pressure acutely rises during the first hours of both ischemic and hemorrhagic stroke and that hypotension during the acute stroke is severely detrimental. The injured brain has less tolerance for decreased perfusion, and the mechanisms of autoregulation do not function as well. The current recommendations from the American Heart Association (AHA) and the Anesthesia Society of America (ASA) are echoed throughout the literature. In a patient who is

eligible for thrombolytic treatment, blood pressure should be lowered to an upper threshold of 185/110 before the treatment because poorer outcomes due to bleeding have been associated with thrombolytic administration in patients with higher pressures. Pressure reduction can be achieved by using either labetalol, 10 to 20 mg IV over 1 to 2 minutes, repeated once, or nicardipine infusion, 5 mg/hour, titrated up by 2.5 mg/hour, to a maximum of 15 mg/hour. Transdermal nitroglycerin (1 to 2 inches) is a third option. If blood pressure cannot be controlled with these initial medications, thrombolytic use is not indicated because of the higher risk of intracerebral bleeding. Once thrombolytic therapy is started, blood pressure must be maintained below 180/105 mm Hg for the next 24 hours to minimize the risk of bleeding. Patients with an ischemic stroke who are not eligible for thrombolytics should not have their blood pressure reduced, unless it is higher than 220/120, in order to maintain cerebral perfusion. One caveat is that blood pressure may need to be reduced because of concomitant MI, acute heart failure, aortic dissection, or hypertensive encephalopathy. In these patients, the systolic blood pressure should be lowered carefully up to 15% while monitoring for worsening neurologic changes.7 Blood pressure control for a spontaneous intracerebral hemorrhage (SIH) is managed differently. Unlike trials in ischemic stroke that revealed less favorable outcomes for patients when their blood pressure was either very high or modestly low, hemorrhagic stroke appears to not be worsened by quickly lowering the blood pressure in the early hours of stroke.8 As a result, the current AHA/ASA guidelines are to lower the blood pressure quickly on presentation. The current consensus recommendations are based on the suspicion of increased ICP.9 If the systolic blood pressure is higher than 180 or the MAP is higher than 130 31


Critical Decisions in Emergency Medicine / Neurologic Edition

mm Hg and there are no signs of elevated ICP, then consider a target of 160/90 or a MAP of 110. This should be achieved by intermittent intravenous medications, and the patient should be reassessed every 15 minutes. If systolic blood pressure is higher than 180 or the MAP is higher than 130 mm Hg and there is a possibility of elevated ICP, then blood pressure reduction should be managed with intermittent or continuous intravenous agents only after intracranial pressure monitoring has been established. The goal is to keep the cerebral perfusion pressures greater than or equal to 60 mm Hg. If the patient’s systolic blood pressure is higher than 200 or MAP is higher than 150, aggressive reduction with continuous intravenous infusion should be started. ICP should be monitored with the same goal of keeping the cerebral perfusion pressure at or above 60 mm Hg. When assessing the patient for increased ICP or risk for aspiration, the physician should also make a decision about head positioning. A midline position is preferred, and if any trauma is suspected, a cervical collar is warranted. The decision to elevate the head of the bed in the acute stroke patient should be made in relation to the patient’s systolic blood pressure and balanced against the concern for increased ICP. Elevating the head of the bed can decrease ICP and, in a hypotensive patient, worsen the stroke. In a patient with an elevated ICP from intraparenchymal hemorrhage, a large ischemic stroke, or a mass lesion, elevating the head of the bed 30° can help reduce the ICP. If the patient is at risk for aspiration or has concomitant heart failure, then it also prudent to elevate the patient’s head to maintain oxygenation. Dysphagia can occur in the stroke patient; perform a swallowing test prior to administering any oral medications. Fever should be controlled, but there is no current evidence supporting induced hypothermia in stroke patients. The

administration of aspirin within the first 24 to 48 hours has been shown to help reduce the progression of ischemic stroke without significantly increasing the incidence of hemorrhagic transformation and should be implemented as long as the patient does not receive thrombolytics.10 If the patient does receive thrombolytics, aspirin should not be started until 24 hours after thrombolytic therapy is stopped.10 CRITICAL DECISION Which imaging modality is most useful in the evaluation of a stroke patient?

With a thorough history and evaluation, a stroke can be reliably diagnosed clinically without imaging studies. The purpose of imaging is to classify a stroke as hemorrhagic or ischemic and to help in directing therapy. Initial imaging results in ischemic stroke are often normal, making the decision to give thrombolytics clinical. It is of utmost importance to detect a hemorrhagic stroke, in which thrombolytic therapy would be devastating. NECT scanning remains the most widely used and accepted initial imaging study on the stroke patient. NECT is recommended by the AHA, ASA, and the National Institute of Neurologic Disorders and Stroke (NINDS). Current guidelines suggest a door-to-NECT time of less than 25 minutes and a doorto-imaging-interpretation time of less than 45 minutes for all patients with suspected stroke.7 NECT can reliably detect hemorrhagic strokes, large areas of infarction in ischemic strokes, and other intracranial lesions. These findings are important because they represent contraindications to thrombolytic use. It is recommended that thrombolytics be withheld in patients with very large ischemic strokes that involve more than onethird of the area fed by the MCA. Using thrombolytics in these patients carries a greater risk of bleeding.7 Magnetic resonance imaging (MRI) has also proved useful in the

initial imaging of stroke patients. It is just as sensitive for revealing bleeding, tumor, abscesses, and large area infarcts and is included in the guidelines as an equal option for initial imaging. Concurrent noninvasive angiography, either by CT or MRI (CTA, MRA), can help certain patients with large clots who might benefit from intra-arterial thrombolytics or mechanical clot extraction but is not the standard of care and is not widely available (discussed below). It can also offer predictive information, suggesting a poorer prognosis if large proximal or cerebellar occlusions are found. The 2013 guidelines for management of stroke offer a new recommendation to include perfusion-weighted imaging (either CT or MR) in select patients. Current literature shows that even after the 4.5-hour window for thrombolytics has expired, an area of ischemic yet viable tissue can sometimes remain surrounding the infarcted core. This area of viable tissue, known as the penumbra, may be salvageable by intra-arterial thrombolytics or mechanical clot extraction. Perfusionweighted imaging can show the presence or absence of this ischemic penumbra. It should be remembered that perfusion weighted imaging adds both time and, in the case of CT, additional radiation. The imaging should only be performed if the institution has the ability to perform interventions based on the information gained from the imaging. New to the national guidelines is approval of transcranial Doppler ultrasonography for monitoring development of arterial vasospasm; this can be implemented in the emergency department and continued in the ICU setting.11 CRITICAL DECISION How should thrombolytic therapy be implemented in patients with acute ischemic stroke?

In 1996, the US Food and Drug Administration (FDA) approved the

32


Critical Decisions in Emergency Medicine / Neurologic Edition

use of intravenous tissue plasminogen activator (tPA) for ischemic stroke presenting within 3 hours of onset based of results from the NINDS II trials.12 This approval recommended 0.9 mg/kg of tPA based on actual body weight. Ten percent of the total dose should be given as an initial bolus over 1 minute, and the remaining 90% given over 60 minutes. The maximum dose should not exceed 90 mg. Currently the AHA/ASA, the American Academy of Neurology (AAN), and the American College of Emergency Physicians

(ACEP) have published similar clinical policy guidelines endorsing the use of tPA in ischemic stroke.7,13 The most recent guidelines, based on the ECASS III trial, include the use of tPA for a subgroup of the stroke population that presents within the 3 to 4.5 hour window from known symptom onset. As of this writing, the FDA has not approved the use of tPA for this extended window, although its use has been approved by the European Medicines Agency. The inclusion and exclusion criteria for the use of tPA (Table 2)

are derived from both the drug’s mechanism as well as results from the various trials. Many hospital systems use an initial NIHSS score of 4 or less as an indicator of a “minor stroke” and therefore exclude these patients from tPA administration. The most recent recommendations from the AHA/ASA suggest that patients with NIHSS less that 4 may benefit from tPA. It is possible to score relatively low (2, for example) with either an isolated gait disturbance, hemianopia, or aphasia. Any of these disabilities could easily be considered more than

Table 2. Inclusion and Exclusion Criteria: Administration of Thrombolytic Therapy Inclusion Criteria: Patients presenting within 3 hours of symptom onset or last seen normal Diagnosis of an ischemic stroke causing measurable neurologic deficit Onset of symptoms is known and is within 3 hours of the beginning of treatment The patient is 18 years of age or older Exclusion Criteria: Patients presenting within 3 hours Significant head trauma or prior stroke in the previous 3 months Symptoms suggest subarachnoid hemorrhage Arterial puncture at a noncompressible site in the previous 7 days History of previous intracranial hemorrhage Intracranial neoplasm, arteriovenous malformation, or aneurysm Recent intracranial or intraspinal surgery Elevated blood pressure that remains higher than 180 mm Hg systolic or above 110 diastolic mm Hg Active internal bleeding Acute bleeding diathesis, including: Platelet count below 100,000/mm3 Heparin received within last 48 hours that leads to an elevated aPTT Current use of warfarin with an INR above 1.7 or PT longer than 15 seconds Current use of direct thrombin inhibitors or direct factor Xa inhibitors with elevated sensitive laboratory tests (such as aPTT, INR, platelet count, and ecarin clotting time; TT or appropriate factor Xa activity assays) Blood glucose less than 50 mg/dL CT-proven area of multilobar infarct larger than 1/3 total cerebral hemisphere Relative Exclusion Criteria: Patients presenting within 3 hours Minor or rapidly improving stroke symptoms Pregnancy Seizure at the onset with postictal residual neurologic impairments Major surgery or serious trauma within previous 14 days Recent gastrointestinal or urinary tract bleeding in the past 21 days Acute MI in the past 3 months Inclusion Criteria: Patients presenting 3 to 4.5 hours from symptom onset or last seen normal Diagnosis of an ischemic stroke causing a measurable neurologic deficit Relative Exclusion Criteria: Patients presenting from 3 to 4.5 hours from onset or last seen normal Age above 80 years Severe stroke with NIHSS score of more than 25 Patients taking an oral anticoagulant regardless of INR Patients with history of both diabetes and prior ischemic stroke

33


Critical Decisions in Emergency Medicine / Neurologic Edition

“minor” in the context of a patient’s other disabilities, socioeconomic factors, and perception of disease. Unfortunately, wasted time or improper assumptions can make the patient temporally ineligible for treatment, and time of symptom onset should be one of the first facts verified from the EMS personnel at patient delivery. The discussion with the patient and family about thrombolytic therapy is similar to consent discussions involving other potentially harmful interventions. It is helpful to provide the abbreviated reasoning behind the recommendation to give tPA and discuss the risks of increased morbidity and death. The discussion should also include the likelihood of improvement without administering tPA. Like all decisions involving an intervention with substantial risk to the patient, involving the patient and the family in the discussion should be well documented. The emergency physician should discuss the use of tPA with the neurologist or neuro-interventionalist (if available at the treating institution) as early as possible in the care of the stroke patient. Despite a clear recommendation for the use of tPA in acute ischemic stroke patients, its use in nonstroke centers remains low. A clinical trial not yet published is looking at stroke care provided in community emergency departments without early access to specialists.14 Emergency physicians must understand that tPA is an approved treatment for acute ischemic stroke, and consent, although recommended, is not necessary when it cannot be obtained in a timely manner. Also the administration of intravenous tPA to a patient who is not suffering from a stroke but instead is presenting with a stroke mimic (seizure, sepsis, etc.) has been extensively studied. It carries with it no known increased mortality risk providing the patient has no contraindications to tPA (Table 2).

CRITICAL DECISION How should emergency physicians manage complications from thrombolytic use?

Once started on tPA, patients should be frequently observed for complications from the thrombolytic as well as for worsening symptoms and complications from the initial neurologic insult. If the patient develops signs of ICH such as an acute increase in blood pressure, sudden headache, nausea, or vomiting or has new findings on repeat neurologic examinations, the tPA should be discontinued and a repeat NECT of the head should immediately be obtained. The ABCs should be monitored and the patient intubated as necessary. If ICP elevation is suspected, the head of the bed should be elevated to 30°. Cryoprecipitate, fresh frozen plasma (FFP), and platelets are often administered. Tranexamic acid, a plasminogen activator, has reportedly been used but is not FDA approved. Blood tests should be sent for type and crossmatching, prothrombin time (PT), partial thromboplastin time (PTT), and thrombin time (TT). Surgical consultation should be sought early. Minor bleeding such as gingival bleeding, oozing from catheter sites, and bruising does not require stopping tPA, but does require local care and close monitoring. Finally, angioedema occurs infrequently as a result of tPA therapy. Usually it is minor and resolves; however, if the angioedema is severe, tPA must be stopped and standard treatments consisting of steroids and antihistamines should be initiated while RSI is considered. CRITICAL DECISION What is the role of endovascular intervention in the treatment of stroke?

The benefit of endovascular intervention for SAH is well described and is the current best practice in most cases. Conclusive data are lacking for its benefit in cases of ischemic stroke, although many

trials are ongoing. Management of SAH is beyond the scope of this lesson; however, clipping, coiling, and endovascular stent placement for definitive treatment can be used by the neurosurgeon or the neuro-interventionalist.11 With regards to ischemic stroke, a recent study showed no difference in outcomes between the use of intravenous tPA alone and the use of intravenous tPA followed by endovascular intervention.15 However, the frequency of anecdotal success from mechanical clot extraction remains puzzling as patients with significant deficits have been reported to experience marked and rapid recoveries directly following large vessel clot extraction. Prevailing arguments for intravascular intervention purport that the most recent studies showing no benefits were designed several years before appropriate patient selection criteria were determined. Newer devices may also yield better outcomes. The current options under investigation and in practice at major stroke centers include both localized intra-arterial tPA, with or without standard intravenous tPA, and mechanical clot disruption via a variety of clot retrieval, aspiration, or stenting devices. Emergency physicians should know what tools are currently being explored and implemented at their institutions or at their receiving facilities to best provide proper disposition of their patients within appropriate time frames and to know which specialists are needed for consultation. Despite the continued controversy between the effectiveness of tPA compared to the use of intravascular techniques, interventional therapy for clinically significant large vessel occlusions (traditionally, NIHSS score above 8) when intravenous tPA has been unsuccessful during the first hour may hold some promise. ACEP does not have a current clinical policy addressing the various endovascular options. However, both the AHA/ASA and the American 34


Critical Decisions in Emergency Medicine / Neurologic Edition

College of Chest Physicians (ACCP) offer some clinical guidelines.7,16 Both guidelines agree that it is reasonable to attempt catheter-directed intraarterial tPA in patients who present within 6 hours of stroke onset as long as they have contraindications to the standard intravenous tPA. Of these patients, the ASA further states that the selected patients should have large occlusions, specifically their MCA. Although both guidelines state that intravenous tPA is preferred to a combination of intravenous and intraarterial tPA, the ASA guidelines state that intra-arterial tPA and mechanical devices are reasonable options as “rescue” treatments if intravenous tPA has already been attempted. Failure of intravenous tPA is not clearly defined in either guideline, but the ASA guidelines cite a small, prospective cohort trial with good outcomes and no increased rate of complications when the rescue intervention was done after a 1-hour trial of intravenous tPA with no response. The guidelines differ with respect to intra-arterial mechanical clot disruption. The ACCP simply states that mechanical clot disruption is not recommended for ischemic strokes, with a small caveat that some carefully selected patients may choose the unproven benefits over the associated risks. The ASA guidelines, published a year later, in 2013, state that mechanical thrombectomy or a rescue device is reasonable in patients with contraindications to intravenous tPA, although more trials are needed to address specific timelines and patient populations. Neither the ACCP nor ASA offer endorsement of intracranial angioplasty or stenting outside the setting of clinical trials. CRITICAL DECISION What is the appropriate emergency department management of stroke caused by SIH?

Initial management of a patient with suspected SIH includes attending to the ABCs and rapid imaging. Once SIH is confirmed, emergency physicians must determine if any

interventions are warranted early in the patient’s course and decide on the proper disposition. The incidence of SIH is increasing in the United States, in part because of the use of oral anticoagulants for conditions such as chronic atrial fibrillation, deep venous thrombosis, and heart valve disease. Patients on anticoagulants represent a special population because the bleeding could be a result of the anticoagulant. Current recommendations are to reverse any coagulopathies.9 SIH in a patient on antiplatelet medications such as aspirin, clopidogrel or dipyridamole/aspirin should receive both supplemental platelets and desmopressin. Patients on warfarin have a deficiency of the vitamin K dependent factors (II, VII, IX, and X) and should be given both vitamin K and FFP or prothrombin complex concentrates (PCCs) containing these missing factors or activated factor VII. The routes and doses of each are directed by the patients’ status and INR at the time of presentation and are often institution specific. Heparin reversal with protamine is appropriate. Although enoxaparin is more difficult to reverse, patients on this medication should also be treated with protamine. Dabigatran and rivaroxaban are newer direct thrombin inhibitors that are becoming more popular as anticoagulants because of their simple dosing and relatively steady state in the blood stream allowing for administration without coagulation monitoring. They act well down stream of the vitamin K dependent factors but have no effect on platelets, making reversal difficult with currently available agents. Treatment options are theoretical as no studies have yet been published. Patients’ activated PTT and TT should be monitored and treatment considered if these times are prolonged. FFP, PCC, and activated factor VIIa all contain small amounts of thrombin and theoretically could be used in the reversal of the coagulopathy.

Thrombin inhibitors are also absorbed by activated charcoal in the first 2 hours after administration and are amenable to removal by dialysis, making these untested options in the bleeding patient.17 Continuous ICU monitoring and surgical consultation for possible ICP monitor placement are required for all SIH patients. Options for the rarer, noncoagulopathic SIH include cerebrospinal fluid drainage or surgical hematoma evacuation. Early consultation and intervention are paramount. If there are signs of increased ICP such as papilledema or hypertension with bradycardia (Cushing reflex), which generally precedes uncal herniation (ipsilateral dilated pupil and contralateral paralysis), or if there is evidence of bleeding seen on the CT, the head of the patient’s bed should be elevated to 30° and temporizing measures using osmotic diuretics such as mannitol, 1 g/kg, or hypertonic saline should be initiated. RSI, neuromuscular blockade, and sedation should be used. New guidelines suggest that patients with a Glasgow Coma Scale score of less than 8, those with clinical evidence of transtentorial herniation, and those with significant intraventricular hemorrhage or hydrocephalus be considered for ICP monitoring and treatment.11 A cerebral perfusion pressure of 50 to 70 mm Hg should be maintained. Seizures should be treated with standard anticonvulsant drugs, although prophylactic anticonvulsant medication is no longer recommended. Performing an EEG on those patients with decreased level of consciousness to monitor for subclinical seizures is also suggested.11 CRITICAL DECISION What is the appropriate disposition for stroke patients after treatment in the emergency department?

Transfer to a stoke center or admission to a neurology ICU or stroke unit capable of specialized stroke care is prudent. These

35


Critical Decisions in Emergency Medicine / Neurologic Edition

patients require frequent neurologic assessments by providers familiar with both the complications of stroke and the medications used. Intravenous tPA administration requires that the patient receive neurologic examinations every 15 minutes for 2 hours, every 30 minutes for 6 more hours, and then every hour until 24 hours have elapsed. Frequent blood pressure monitoring is essential, with a target pressure of less than 180/105 mm Hg. Patients who are intubated or have had an intervention by the neuro-intensivist should be admitted to a neurology ICU. Ischemic stroke carries a 25% chance of continued deterioration; of those who deteriorate, increasing cerebral edema is causative in 33%.7 Although, it usually presents 3 to 4 days after the onset of symptoms, severe large-volume edema, aptly termed malignant edema, can occur within the first 24 hours of symptom onset. Large volume and cerebellar

ischemic strokes, especially, require frequent neurologic evaluations because of the potential for rapid deterioration from the edema; these patients could require emergent craniotomy to allow for edema progression. Stroke patients suffering from SIH are at risk for the same complications of progressive edema as ischemic stroke patients. In addition, the possibility of intraventricular hemorrhage exists, which can lead to obstruction of cerebrospinal fluid flow, hydrocephalus, and resultant increased ICP. Current guidelines suggest that placing a ventricular drain can be therapeutic; neurosurgical consultation should be sought.9 Ongoing studies are examining the possibility of thrombolytic use through the ventricular drain, therapeutic lumbar puncture, and surgical clot evacuation, although these interventions are not currently

recommended. All stroke patients are at risk for seizures, hypotension, fever, and falls. Fluctuations in sodium and glucose levels can occur. Appropriate ancillary tests and laboratory studies should be obtained (Table 3). Ideally, many of these should be performed in the emergency department and then continued on the appropriate unit once the patient is admitted. Although seizure prophylaxis is not recommended, treatment of clinically diagnosed seizures and surveillance with continuous electroencephalography for subclinical seizures in obtunded patients are appropriate interventions.

Case Resolutions â– Case One The 58-year-old man with rightsided weakness and difficulty speaking was a candidate for tPA if his blood pressure could be reduced.

Table 3. Ancillary Tests and Laboratory Studies Useful in the Evaluation and Management of Stroke Assay ECG and cardiac monitoring Pulse oximetry CBC with differential

Point-of-care glucose testing

Electrolytes Cardiac enzymes Coagulation studies Urinalysis Pregnancy test Toxicology screen Lumbar puncture

Rationale Atrial fibrillation is associated with embolic stroke. Arrhythmia can be associated with hypoperfusion and mimic stroke. Hypoxia can mimic stroke. Hypoxia in the setting of stroke worsens outcomes. Severe anemia can mimic stroke. Anemia in the setting of stroke worsens outcomes. Polycythemia/thrombocytosis or thrombocytopenia can implicate source of stroke or indicate the possibility of coagulopathy. Hypoglycemia is a common stroke mimic. Stroke in the setting of hypoglycemia has worse outcomes. Stroke in the setting of hyperglycemia has worse outcomes. Hyponatremia, hypocalcemia can mimic stroke. Troponin can elevate during stroke independent of MI processes. Stroke is a common complication of MI. Coagulopathy is a contraindication to thrombolytics. Coagulopathy needs to be corrected in hemorrhagic strokes. Infection can mimic stroke in the very sick and elderly. Infection in the setting of stroke will worsen overall outcome and requires treatment. Pregnancy is not an absolute contraindication to thrombolytics but should be part of the informed consent discussion. Drugs of abuse can mimic stroke and cloud the clinical picture. Meningitis and encephalitis can mimic stroke; this test is invaluable in the right clinical setting. However, performing an LP on a patient will make them ineligible for subsequent tPA administration.

36


Critical Decisions in Emergency Medicine / Neurologic Edition

He was started on a nicardipine drip, beginning at 5 mg/hour and titrated up by 2.5 mg/hour, and was reassessed frequently. The on-call neurologist, the emergency physician, and the patient’s family discussed the recommendation to administer tPA when the patient’s blood pressure was within a safe range, and consent was obtained. When the patient’s blood pressure was below 185/110, he was given intravenous tPA. In the first hour of treatment, while awaiting transfer to the neurology ICU, the patient’s symptoms did not improve, but he remained hemodynamically stable and without deterioration. He was transferred to the neurology

ICU where frequent neurologic examinations showed slow but steady improvement in symptoms over the first 24 hours. By the end of 48 hours he could give a “thumbs-up” with the previously affected hand. Intensive rehabilitation was started and the patient was discharged from the hospital to an outpatient rehabilitation center for stroke less than a week later.

■ Case Two

The patient with a headache who became unresponsive had taken his last dose of dabigatran an hour prior to presentation. He was given activated charcoal via nasogastric

Pearls • Strokes caused by hypoperfusion can present initially as focal strokes rather than with bilateral deficits because of disproportionate unilateral atherosclerosis. • Thrombolytics are an approved therapy for ischemic strokes; consent should be obtained if possible. However, if the patient is obtunded or otherwise unable to provide consent and if surrogate decision makers are unavailable, this life-saving intervention should be performed in appropriate candidates. • Administering intravenous tPA to a patient who is not suffering from a stroke but instead is presenting with a stoke mimic (eg, seizure, sepsis) carries with it no known increased mortality risk, providing the patient is not actively bleeding. • The administration of tPA does not make the patient ineligible for subsequent intravascular intervention including intra-arterial directed tPA and mechanical thrombectomy; seek neurosurgery or neurointerventional specialist consultation regarding these interventions early. • Patients with an ischemic stroke who are not candidates for tPA should not have their blood pressure reduced unless it is above 220/120 unless other clinical features require reduction; the increased pressure can help maintain cerebral perfusion.

Pitfalls • Failing to ensure that there is appropriate communication with families and specialists; thrombolytics used to treat stroke carry with them an intrinsic risk that may not be obvious to patients or their families. • Overlooking an EMS report or failing to pursue information about the timing of symptom onset; this could result in wasted time or an improper assumption that a patient is ineligible for tPA. • Failing to admit stroke patients to special nursing units or ICUs with the appropriate capabilities; standard floor beds, which may have monitors, are not appropriate dispositions for these patients.

tube while emergent dialysis in the emergency department was arranged. The head of his bed was elevated 30°. He was given FFP, activated factor VII, and mannitol. Blood pressure was initially lowered with a nicardipine infusion, and neurosurgery was consulted for placement of an ICP monitor or possible decompressive craniectomy. The patient still showed no signs of resolution of his elevated ICP. The patient’s bleed was determined by neurosurgery to be very deep, and no significant improvement was expected from surgical drainage, but the neurosurgeon thought it prudent to perform a decompressive craniotomy. The patient hemodynamically stabilized after surgery, and subsequent NECT showed stabilization of the bleed with some resolution 24 hours later. He was eventually discharged to an outpatient rehabilitation facility.

Summary The appropriate diagnosis of stroke in the emergency department is achieved by the diligent emergency physician’s focused history and physical examination. Expedient diagnostic imaging to differentiate intraparenchymal hemorrhage from ischemic stroke and early contact with neurology should be in motion as any delay can easily place a patient outside the window for treatment. The mimics of ischemic stroke are common and are so named because of their indistinguishable presentations at the bedside. The ABCs of emergency medicine have highest priority for any stoke patient with special blood pressure goals. In each case, the patient’s best odds for recovery will be determined by the emergency physician’s ability to act swiftly, communicate clearly with consultants, and apply the appropriate therapy as early as possible. Research is ongoing in the field of both stroke diagnosis and treatment thus requiring emergency physicians to stay abreast of the rapid influx of new information. 37


Critical Decisions in Emergency Medicine / Neurologic Edition

References 1. Go AS, Mozzaffarian D, Roger VL, et al. Heart disease and stroke statistics—2013 update: a report of the American Heart Association. Circulation. 2013;127(1):e6-e245. 2. Marler JR, Tilley BC, Lu M, et al. Early stroke treatment associated with better outcome: the NINDS rt-PA stroke study. Neurology. 2000;55(11):1649-1655. 3. Bamford J, Sandercock P, Dennis M, et al. Classification and natural history of clinically identifiable subtypes of cerebral infarction. Lancet. 1991;337 (8756):1521-1526. 4. Brott T, Adams HP Jr, Olinger CP, et al. Measurements of acute cerebral infarction: a clinical examination scale. Stroke. 1989;20(7):864-870. 5. Robinson N, Clancy M. In patients with head injury undergoing rapid sequence intubation, does pretreatment with intravenous lignocaine/lidocaine lead to an improved neurological outcome? A review of the literature. Emerg Med J. 2001;18(6):453-457. 6. Lanier WL, Iaizzo PA, Milde JH. Cerebral function and muscle afferent activity following intravenous succinylcholine in dogs anesthetized with halothane: the effects of pretreatment with a defasciculating dose of pancuronium. Anesthesiology. 1989;71(1):8795. 7. Jauch EC, Saver JL, Adams HP Jr, et al. Guidelines for the early management of patients with acute ischemic stroke : a guideline for healthcare professionals from the American Heart Association/American Stroke Association. Stroke. 2013;44(3):870-947. 8. Qureshi AI. Antihypertensive Treatment of Acute Cerebral Hemorrhage (ATACH): rationale and design. Neurocrit Care. 2007;6(1):56-66. 9. Morgenstern LB, Hemphill CJ III, Anderson C, et al. Guidelines for the management of spontaneous intracerebral hemorrhage: a guideline for healthcare professionals from the American Heart Association/ American Stroke Association. Stroke. 2010:41(9):21082129. Available online at: http://stroke.ahajournals. org/content/41/9/2108. 10. International Stroke Trial Collaborative Group. The International Stroke Trial (IST): a randomised trial of aspirin, subcutaneous heparin, both, or neither among 19435 patients with acute ischaemic stroke. Lancet. 1997;349(9065):1569-1581. 11. Connolly ES Jr, Rabinstein AA, Carhuapoma JR, et al. Guidelines for the management of aneurysmal subarachnoid hemorrhage: a guideline for healthcare professionals from the American Heart Association/ American Stroke Association. Stroke. 2012;43(6):17111737. 12. National Institute of Neurological Disorders and Stroke rt-PA Stroke Study Group. Tissue plasminogen activator for acute ischemic stroke. N Engl J Med. 1995;333(24):1581-1587. 13. American College of Emergency Physicians, American Academy of Neurology. Clinical Policy: Use of intravenous tPA for the management of acute ischemic stroke in the emergency department. Ann Emerg Med. 2013;61(2):225-243. 14. Scott PA, Xu Z, Meurer WJ, et al. Attitudes and beliefs of Michigan emergency physicians toward tissue plasminogen activator use in stroke: baseline survey results from the INcreasing Stroke Treatment through INteractive behavioral Change Tactic (INSTINCT) trial hospitals. Stroke. 2010;41(9):2026-2032. 15. Broderick JP, Palesch YY, Demchuk AM, et al. Endovascular therapy after intravenous t-PA versus t-PA alone for stroke. N Engl J Med. 2013;368(10):893903. 16. Lansberg MG, O’Donnell MJ, Khatri P, et al. Antithrombotic and thrombolytics therapy for ischemic stroke: antithrombotic therapy and prevention of thrombosis, 9th ed: American College of Chest Physician Evidence-Based Clinical Practice Guidelines. Chest. 2012;141(2 Suppl):e601S-e636S. 17. van Ryn J, Stangier J, Haertter S, et al. Dabigatran etexilate—a novel, reversible, oral direct thrombin inhibitor: Interpretation of coagulation assays and reversal of anticoagulant activity. Thromb Haemost. 2010;103(6):1116-1127.

38


Critical Decisions in Emergency Medicine / Neurologic Edition

31. A 55-year-old man presents with sudden onset of vomiting, decreased level of consciousness, confusion, visual field deficits, and right-sided weakness. He is hypertensive, with occasional tachypnea and long apneic pauses as well as oxygen desaturation requiring intubation. His wife states he complained of a headache just before the onset of his symptoms that was not relieved when he took his morning blood pressure medications. Which of the following findings on brain imaging best matches the patient’s clinical findings? A. Lacunar infarct in the left internal capsule with left carotid plaques B. Lacunar infarct in the right internal capsule with right carotid plaques C. Left posterior cerebral artery occlusion with evidence of prior vessel disease D. Left temporal-parietal intraparenchymal hemorrhage with mass effect 32. Which section of the NIHSS cannot be scored for the intubated patient and is therefore recorded as “UN?” A. Best language B. Dysarthria C. Extinction and inattention D. Level-of-consciousness questions 33. A comatose 70-year-old man is brought in by EMS after a rapid decline in mental status that began with a profound right-sided hemiparesis. He is now outwardly unresponsive and is intubated without paralytics or sedatives. During NIHSS scoring, a “3” is given during the visual field testing due to an absence of response to visual threat in all fields. A “3” is also scored during facial palsy testing as the patient failed to respond to a noxious stimulus. When testing for a gaze palsy, how should the patient be assessed? A. Automatic scoring of 0 for inability to test B. Automatic scoring of 2 for unresponsiveness C. Caloric testing in either ear and scoring according to nystagmus response D. Manual rotation of the patient’s head, scoring based on presence of oculocephalic reflex 34. The rationale behind the recommendation for nonenhanced CT (NECT) as the primary imaging mode for stroke patients is that it is available in most hospitals, it can be rapidly implemented and interpreted, and which of the following? A. It can detect intracranial hemorrhage B. It can detect small- to moderate-sized areas of infarction C. It can visualize ischemic penumbra D. It can visualize small cerebellar and brain stem lesions

36. A 57-year-old man presents within 2 hours of symptom onset with an ischemic stroke. The family decides after careful discussion with the emergency physician and neurologist that tPA should be administered. Which of the following features would allow him to receive tPA? A. Active internal bleeding B. Hypertension with a blood pressure of 175/95 C. Previous intracranial hemorrhage D. Significant head trauma 3 months ago 37. A 64-year-old woman with diabetes mellitus, hypertension, and gout presents via EMS with left facial weakness, right hemiparesis, and right-sided paresthesias. Her blood pressure is 178/98 and the rest of her vital signs are normal. Which should be completed before tPA is administered? A. Bedside glucose B. Elevation of the head of the bed to 30° C. Reduction of blood pressure to 160/80 D. Serum phosphorus level 38. A 70-year-old man presents with an apparent ischemic stroke 3.5 hours after symptom onset and a negative NECT. Which would absolutely exclude him from eligibility for treatment with tPA? A. History of an arteriovenous malformation B. Prior history of diabetes and ischemic stroke C. Severe stroke with NIHSS score above 25 D. Use of an oral anticoagulant regardless of INR 39. A 72-year-old woman presents with severe vomiting and altered mental status. She has had an aortic valve replacement and is on warfarin. A large intraparenchymal left parietal hemorrhage is seen on the NECT. Which would not be appropriate in the management of this patient? A. FFP B. Intravenous vitamin K C. PCC D. Protamine 40. A 35-year-old woman presents with dysphagia, dyscalculia, and ipsilateral motor and sensory deficits in the left face and upper extremity. Her symptoms started 2 hours ago, and the NECT reveals no hemorrhage. She should not receive tPA if: A. She had a seizure at the onset with postictal residual neurologic impairment B. She had a small subdural hematoma from a car accident 1.5 months ago C. She had minor surgery within the previous 14 days D. She is pregnant

35. A 45-year-old professional piano player presents after 5 hours of left-arm paralysis that began suddenly during a practice session and was accompanied by left-sided facial droop and paresthesia in the left leg. Which of the following supplemental imaging modalities could be appropriately implemented, as positive findings could lead to intervention even this far outside the classic treatment window? A. CT with perfusion scanning B. MRI C. NECT D. Transcranial Doppler via bedside ultrasound

39


Turn static files into dynamic content formats.

Create a flipbook
Issuu converts static files into: digital portfolios, online yearbooks, online catalogs, digital photo albums and more. Sign up and create your flipbook.